Вы находитесь на странице: 1из 149

www.gradeup.

co

1
www.gradeup.co
CHAPTER-1

Diode Circuits ……..……………….………………………………….………. 4-21


Answer Key …………………………………………………………………………………………………………………..12
C
O
Solution …………………………………………………………………………………………………………………... 12-21

CHAPTER-2

Voltage Regulators ………….……………….………………………….. 23-29


Answer Key ………………………………………………………………………………………………………..……... 26
Solution ………………………………………………………………………………………………………………... 26-29

N
CHAPTER-3

BJT Biasing & Stabilization …………………………………………. 31-42


Answer Key ……………………………………………………………………………………………………….………. 35
Solution …………………………………………………………………………………………………………..……. 36-42

CHAPTER-4

T
Current Mirror Circuits ……..……………………………………….. 44-49
Answer Key ………………………………………………………………………………………………………..………. 46
Solution ……………………………………………………………………………………………………………….. 46-49

CHAPTER-5

E
BJT Amplifiers ………………………………………………..………………. 51-63
Answer Key ………………………………………………………………………………………………………..………. 55
Solution ……………………………………………………………………………………………………………….. 56-63

CHAPTER-6

JFET Biasing & Amplifiers (Only ESE) ……………………….. 65-69

N
Answer Key ………………………………………………………………………………………………………..………. 66
Solution ………………………………………………………………………………………………………………... 67-69

CHAPTER-7

MOSFET Biasing & Amplifiers …………………………………….. 71-86


Answer Key ………………………………………………………………………………………………………..………. 77
Solution ………………………………………………………………………………………………………………... 77-86

2
T
www.gradeup.co
CHAPTER-8

Multi-Stage Amplifiers ….….………………………………….……. 90-97


Answer Key ………………………………………………………………………………………………………………… 93
C
O
Solution ……………………………………………………………………………………………………………….... 93-97

CHAPTER-9

Feedback Amplifiers ……………………………………………….…. 99-105


Answer Key ……………………………………………………………………………………………………….………. 101
Solution ………………………………………………………………………………………………………..……. 102-105

N
CHAPTER-10

Power Amplifiers ……...………….………………………………..…… 106-110


Answer Key ……………………………………………………………………………………………………….………. 107
Solution ………………………………………………………………………………………………………..…….. 108-110

CHAPTER-11

T
Differential Amplifiers …..……………………………………………. 111-114
Answer Key …………………………………………………………………………………………………….…..…..…. 112
Solution …………………………………………………………………………………………………….…….…... 112-114

CHAPTER-12

Operational Amplifiers ………………….……………………..…... 116-137

E
Answer Key ………………………………………………………………………………………………………..…….. 124
Solution ……………………………………………………………………………………………………………... 124-137

CHAPTER-13

Oscillators …………….………………………...…………………………. 138-144


Answer Key ………………………………………………………………………………………………………………. 140

N
Solution …………………………………………………………………………………………………….……….. 141-144

CHAPTER-14

555 Timer & Waveform Generators ……….……………. 145-148


Answer Key ………………………………………………………………………………………………………...……. 146
Solution …………………………………………………………………………………………………………….. 146-148

3
T
www.gradeup.co
Chapter

1 Diode Circuits

1. Assuming diodes are ideal in the figure the If the diode D is ideal, then the transfer
current in diode D1 is___ characteristic curve of the circuit can be
represented as

A.

A. 8 mA B. 5 mA
C. 0 mA D. - 3mA
B.
2. In the given circuit

C.

If V1 = V2 = 0V, silicon diode is to be used


with RF = 35Ω and V  = 0.6 V

The output voltage V0 of the circuit is


______
A. 0.621 V B. 0.738 V D.
C. 0.137 V D. 0 V
3. Consider the circuit shown in the figure
4. The function of the following circuit, if the
below:
input is a sine wave, is that it

4
www.gradeup.co

A. Transmits that part of sine wave, which


is above +8V and below +4V.
B. Transmits that part of sine wave, which
C.
lies between +4V and +8V.
C. Transmits that part of sine wave, which
lies above –4V and below +V.
D. Transmits that part of sine wave, which
lies below +4V and above –8V.
D.
5. For the circuit shown below the input
voltage vi is as shown in figure.

6. For an input Vs = 5 sin(ωt), the circuit


shown in figure below will be behave as
a (Assume that diode is ideal).

A. clipper with sine wave clipped at –2 V


B. clamper with sine wave clamped at +2 V
C. clamper with sine wave clamped at 0 V
Assume the RC time constant large and
D. clipper with sine wave clipped at +2 V
cut-in voltage of diode Vr = 0. The output
7. In a half wave rectifier; if on AC supply is
voltage v0 is
60 Hz, then what is the a.c ripple at
output?
A. 30 Hz B. 60 Hz
A.
C. 120 Hz D. 15 Hz
8. A half-wave rectifier having a resistance
load of 1 kΩ rectifier an a.c. voltage of 325
V peak value and the diode has a forward
resistance of 100Ω. The RMS value of
B.
output current is
A. 295.4 mA B. 94 mA
C. 147.5 mA D. 28 mA

5
www.gradeup.co

9. The components of full-wave voltage A. (1/6) A B. 160 mA


doubler circuit are C. 132 mA D. 154 mA
A. 2 diodes and 1 capacitor 12. Determine Vo in the following circuit.
B. 4 diodes and 1 capacitor Assume all diode to be ideal.
C. 2 diodes and 2 capacitors
D. 4 diodes and 2 capacitors
10. For an ideal diodes P1 and P2, the transfer
characteristics of the circuit will be:

A. Vin B. –Vin
C. 2Vm – Vin D. Vin – 2Vm
13.

A.

The waveform for output vo is,

A.
B.

C. B.

D.
C.

11. If the cut-in voltage and forward resistance


of each diode are 0.7 V and 1 ohm
respectively, the current through 48 ohm
D.
resistor is

14. In the circuit shown below, the three


signals of figure are impressed on the input
terminals.

6
www.gradeup.co

15. Consider the given circuit and a waveform


for the input voltage. The diode in circuit
has cut-in voltage Vγ = 0

If the diode are ideal then the voltage Vo is


The waveform of output voltage v0 is

A.
A.

B. B.

C.
C.

D.
D.

7
www.gradeup.co

16. The load resistance of a center-topped full-


wave rectifier is 500Ω and the necessary
voltage is 60 sin (100 πt). Peak, average
and RMS value of current are respectively A.
(given forward resistance is 50 Ω)
A. 0.077 A, 0.0695 A, 0.109 A
B. 0.032 a, 0.043 A, 0.12 A
C. 0.109 A, 0.0695 A, 0.077 A
D. None of these
B.
17. The i-v characteristics of the diode in the
circuit given below is,

 v − 0.7
 A, v  0.7 V
i =  500
 0 A, v  0.7 V

The current in the circuit is, C.

D.

A. 10.2 mA B. 8.3 mA
19. The minimum and maximum output
C. 5.67 mA D. 6.2 mA
voltage of the given clamping network
18. Let Vγ = 0.7 (diode forward drop), Assume
provided the Vi square wave input is given
the input varies over the range -
to the circuit is:
10 < Vi <10 V.
Plot V0 versus Vi is:

8
www.gradeup.co

A. 38 V and -2 V B. -42 V and 2 V 23. For the following diode circuit, what will be
C. 42 V and 2 V D. -38 V and 2 V the output waveform circuit [Given,
20. In the voltage doubler circuit shown in the Vo (ON) = 0.7 V]
figure, the switch ‘S’ is closed at t = 0.
Assuming diodes D1 and D2 to be ideal,
load resistance to be infinite and initial
capacitor voltages to be zero. The steady
state voltage across capacitor C1 and
C2 will be

A. Vc1 = 10 V, VC2 = 5 V
B. Vc1 = 10 V, VC2 = –5 V
C. Vc1 = 5 V, VC2 = 10 V
D. Vc1 = 5 V, VC2 = –10 V A.
21. In a half wave rectifier the forward value of
resistance of the diode is 30 Ω. The half
wave rectifier was connected to load
RL1 =300 Ω and the efficiency nRL1 was
measured. Now load resistance is
B.
connected to RL2 = 3 KΩ and efficiency was
nRL2. The value of nRL2 – nRL1 is ______%.
A. 4.28 B. 33.2
C. 3.28 D. 3.78
22. The input to full-wave rectifier shown
below is vi = 120 sin 2π60t V. The diode
cut-in voltage is 0.7 V. If the output C.
voltage cannot drop below 100 V, the
required value of the capacitor is ………….
μF.

D.

9
www.gradeup.co

24. Consider the waveform shown below: 26. The transfer characteristics for the clipper
circuit shown below. Assume the diodes
are ideal.

If the input is a 100 V peak square wave


with a period of 20 ms. The output of the
wave is so designed that it has a maximum
value of 35 and minimum of –165 V at the
steady state, then the circuit which can
perform this function can be represented
as.
A.

A.

B.
B.

C.
C.

D. None of these
25. For the ideal diodes as shown, DC
component of output voltage Vo is
______V
D.

27. Assuming the diode to be ideal in the


following circuit, for the output voltage to
be clipped the range of input voltage
should be :

10
www.gradeup.co

A. Maximum input voltage is 12 V


B. DC component of output is –4 V
C. Maximum output voltage is 2 V
D. Minimum output voltage is –13V
31. Consider a clipper circuit as shown in figure
A. Vi > -5V, or Vi < -7 V
below:
B. -7V < Vi < -5V
C. -14 < Vi < -10
D. Vi > -10V or Vi < -14 V
MSQs:
28. While comparing FWR {Fullwave Rectifier}
with Bridge Rectifier which of the following
quantities do not remain same? Here all Diodes are ideal, then

(Keeping input and diode conditions same) A. It input voltage is 5 V, then first Break

A. Form Factor (FF) point occurs.

B. Ripple tailor B. If input voltage is equal to output

C. PIV voltage, then first Break point occurs.

D. Average diode voltage (VD, avg) C. It input voltage is 9 V, output voltage is

29. Circle the correct option(s) for the 3.33 V

following circuit D. If input voltage is 9 V, output voltage is


9 V.
32. Consider the following diode circuit having
cut-in voltage 0.7 V for both diodes, then

A. This circuit is an example of voltage


Tripler
B. VC1 = Vm
C. VC2 = Vm
D. VC3 = 2Vm
30. In the circuit shown, diode is ideal and
input is 3+ 9sinωt then choose the correct
option(s)?
A. ID2 = –0.787 mA
B. ID2 = 0 mA
C. Vo = 10.1 V
D. Vo = 9.53 V

11
www.gradeup.co

ANSWER

1. B 2. A 3. D 4. B 5. B 6. B 7. B 8. C 9. C 10. D

11. C 12. B 13. D 14. B 15. C 16. C 17. D 18. C 19. C 20. D

21. C 22.(20.6) 23. B 24. B 25. (15.11) 26. D 27. D 28. C,D 29. A,B&D 30. A,B&D

31.A,B & D 32. B,D

SOLUTION

1. (B) = 5 - 9.067 × 10-4 × 4.7 × 103


As given diode are ideal, = 5 - 4.262
From the given circuit, it can be observed = 0.738 V
that D2 is OFF and D1 is ON. 3. (D)
∴ D2 = Reverse biased Case I: When Vin(t) > 0
D1 = Forward biased The diode will conduct; thus the equivalent
5 circuit can be given as,
∴ Current through D1 = = 5 mA
1K

2. (B)
If V1 = V2 = 0, both the diodes will be
forward biased &
the equivalent circuit will be

Case 2: When the input voltage Vin < 0.


The diode will not conduct, thus the
equivalent circuit can be given as

I
VCC = IRL + V + (R + R1 )
2 F
I
5 = I  4.7  103 + 0.6 +  (35 + 270)
2
5 − 0.6 R V
I= Vout = Vin = in = 0.5Vin
4700 + 152.5 R +R 2
= 9.067 × 10-4 A
Output voltage, Vout = VCC - IRL

12
www.gradeup.co

13
www.gradeup.co
4. (B) 7. (B)
Vi < 4V, D1 – OFF & D2 – ON For half wave rectifier a.c. ripple at output
⇒ V0 = 4V is equal to input frequency.
For Here, input frequency = 60 Hz
4V < Vi < 8V, D1 – OFF & D2 – OFF ∴ A.C ripple = 60 Hz
⇒ V 0 = Vi 8. (C)
For The peak load current
Vi > 8V, D1 – ON & D2 – OFF 325
Im = = 0.295A
⇒ V0 = 8V (1000 + 100)

5. (B) So, RMS value of current is = 0.295/2

1. When the diode is in downward direction = 147.5 mA

the total signal will be clamp below the 9. (C)

reference voltage.
2. When the diode is in upward direction
the total signal will be clamp above the
reference voltage.
In the given circuit, the diode is in upward
direction and the reference voltage is zero ∴ It requires 2 diodes & 2 capacitors
then the total signal will be clamp above ∴ Option C
the 0V. So, the output voltage is: 10. (D)
For a very small, input voltage, both
D1 and D2 diodes will be OFF because of
12V on cathode side.

6. (B)
The given circuit is,
From figure, V0 = 12 V
For diode D1 to be ON, (Vi > 12V)
Diode D2 will be OFF

This is a clamper circuit,


and the diode is in downward direction the
total signal will be clamp below the
reference voltage.

14
www.gradeup.co
Hence, the transfer characteristics will be,

Here capacitor C1 is charged to voltage Vm

11. (C) with polarity as shown


During negative half cycle D1-on, D2-on,
D3-off, D4-off

As the given source is DC source so

throughout the time only D1 and D4 will be

forward biased. Here capacitor C2 gets charged to voltage


Vm with the polarity as shown
Now the the circuit is
Now to calculate output voltage consider
the circuit as shown below

(8 − 1.4)V 6.6
I= = = 132 mA Apply KVL in the circuit shown
(48 + 1 + 1)  50
Vo = -Vin
12. (B)
13. (D)
Given
Case (I) for Vi < 2V
Diode is ON ∴ equivalent ckt would be,

During positive half cycle D1-off, D2-off,

D3-on, D4-on.
∴ Vo = 2V

15
www.gradeup.co
Case (II) for Vi > 2V 15. (C)
Diode is OFF ∴ equivalent ckt would be CASE I:
If vi + 6 < 0 or vi < –6 diode D1 is OFF and
diode D2 is ON. So, the equivalent circuit is

∴ V o = Vi

So, the output voltage is,


v0 = –6V
CASE II:
If –6V < vi < 8V, then both didoes D 1 and
D2 are OFF. So, the equivalent circuit is
14. (B)
Given circuit diagram

So, the output voltage is


v0 = vi
CASE III:

In this case at output side only that When vi > 8V, then diode D1 is ON and

voltage appear which is maximum in input D2 is OFF. So, the equivalent circuit is,

So
Vo = max (V1, V2, V3)
So Output voltage will be

v0 = 8V
Step 5: From the result obtained in the
above step, we sketch the output
waveform as shown below.

16
www.gradeup.co
i=0.66 mA
then Vo = 10 - 6.66m(10k)
Vo = 3.33 V
Diode will on
if Vi>V0+0.7
Diode will off
if Vi<V0+0.7
16. (C) so
Maximum value of supply voltage, Vsmax = If Vi < 3.33 + 0.7
60V Vi < 4.03 V, Diode will off
Forward resistance, RF = 50Ω ⇒ V0 = 3.33 V

Load resistance, RL = 500Ω if Vi > 4.03, Diode will ON


⇒ V0 = Vi – 0.7 V
Vs max 60
Peak current, Imax = =
RL + RF 500 + 50 19. (C)
The Capacitor would provide the clamping
= 0.109 A
by Vi as it is directly connected to the
2I 2  0.109
Average current, Idc = max = source voltage.
 
Due to the battery, the capacitor would
= 0.0695 A
charge. However, this battery would let the
lmax 0.109
RMS value of current, Irms = = capacitor to charge up to 22 V.( During
2 2
negative cycle)
= 0.077 A During positive cycle of V i,
17. (D) (Vi + VC) = 42 = Vmax and During negative
It is given that for v>0.7 volt cycle of Vi, (-Vi + VC) = 2= Vmin in the
i = (v-0.7)/500 output waveform.
or v = 500i + 0.7 .......(i) 20. (D)
Apply KVL in the given circuit then we get, The given circuit is
-10 + 1x103 (i) + v = 0
or, v = 10-1000 i .........(ii)
From (i) and (ii) we get,
10 – 1000i = 500i + 0.7
1500 i = 9.3
i = 9.3 / 1500 = 0.0062 Step 1: We have the input waveform, vi =
i = 6.2 mA 5 sin wt. So, we draw, the waveform as

18. (C)
Let consider diode is not connected
Apply KVL from +10V to -10V
i=[10-(-10)/(10k+20k)]
i=20/30k

17
www.gradeup.co
Step 2 : For half part of the circuit. When 21. (C)
positive half cycle of input is applied, diode 40.5
% =
R
D1 is ON and D2 is OFF. So, capacitor 1+ f
RL
C1 will charge upto +5 Volt
40.5
VC1 = +5 Volt RL1 = = 36.81
30
1+
300
40.5 40.5
RL2 = = = 40.09
30 1.01
1+
3000
∴ % Increase = 40.09 – 36.81 = 3.28%
22. Ans.
Given the input to full-wave rectifier,
Vi = 120 sin 2π60t V
So, we have the voltage across both the
This is a clamper circuit, So, output of the
secondary transformer as
circuit is
Vs = 120 sin 2π60t V
Therefore, the maximum voltage across
the capacitor is
Vmax = (Vs)max – VD,on = 120 – 0.7 =
119.3V
Since, the input cannot drop below 100 V,
so we have
In this clamper, diode is in downward Vrip = 119.3 – 100 = 19.3V

position. So, it is negative clamper.


Step 3 : Second part of the circuit is peak Thus, the capacitance is obtained as

detector as shown below. Vm 119.3


C= = = 20.6F
2fRVrip 2(60)2.5  103  19.3

23. (B)
Case I : when, vi > 5v
⇒ Diode D1 will be ON and D2 will be OFF.

So, it allows only peaks at the output.


Thus, from the results obtained in the
above step, the output voltage is Vi − 5
 ii = A
VC2 = –10 Volt 5  103

18
www.gradeup.co
Voltage across 3 kΩ resistor, 24. (B)
Vi − 5 Since, the output is reproduced in the
V3k = −3
 3  103 V
5  10 negative cycle, i.e. the clamper is used to

V0 = 5 + 0.6 (Vi – 5) add the negative DC level to the circuit,

Vo = 0.6 Vi + 2 Hence it is a negative clamper.


So, diode must be short circuit for the
Peak value, Vi = 10 V
positive voltage applied at the input.
V0 = 0.6 (10) + 2 = 8 V
Now after deciding the polarity of the diode
Case II: When Vi < - 5 V
we have to make sure that the diode is
→ Diode D1 will be OFF, D2 will be ON
forward biased if the input is lower than 35
Vi + 5
i= A V.
5  103
Thus, the circuit will look like it is given in
Voltage across 3 K Ω resistor: option B figure.
Vi + 5 25. Ans.15.11V
V3k =  3  103 V
5  103 During +ve half cycle D1 ≡ ON, D2 ≡ OFF

Vo = -5 + 0.6 (Vi + 5)
6.8 6.8
Negative peak value of Vo (when Vi = -10 V0 = Vi  = V
6.8 + 2.2 9 i
V);
6.8
Vo = -5 + 0.6 (-10 +5) V0 =  10 sin wt …(i)
9
Vo = -8V
During –ve peak cycle: D2 ≡ ON, D1 ≡ OFF
Case 3:
When -5 V < Vi < 5 V
Diode D1 and D2 will be OFF
⇒ V o = Vi
So, output voltage waveform will be:  6.8 
VO = −Vi   
 6.8 + 2.2 

6.8
V0 = −  10 sin wt …(ii)
9

By (i) & (ii)


Vo = ±7.55 π sin wt
Vm = 7.55 π
2Vm
 VOaVg = = 15.11V

19
www.gradeup.co
26. (D) 40
i= = 1mA
For +ve clipping, diode D1 will be ON and 30 + 10

D2 will be OFF VA = 40 — 10 = 30V


This condition continues till D1 conducts
when Vi≥ 30V
∴ For Vi ≤ 30V D1 – off, D2 – ON and Vo =
30V
So the transfer characteristics is a straight
line with zero slope for Vi ≤ 30V
Vo = -5V
When Vi > 30V
For Diode D1 to be OFF, Vi should be less
D1 is ON
than -5V,
D2 is ON
For, -7V < Vo < -5V
Equivalent circuit diagram
Both diodes D1 and D2 will be OFF

1 V
V0 = Vi = i
1+1 2 Output is same as input
Hence, output is not clipped [Vo = Vi]
Vi 28. (C, D)
i.e. −7   −5
2 Sol.:
⇒ -14 < Vi < -10 (output is not clipped) FWR Bridge Rectifier
So, for clipping input voltage should be
outside to this range FF 1.11 1.11

Vi > -10V or Vi < -14 V Ripple factor 0.483 0.483


27. (D)
When Vi ≤ 0, D1 – off PIV 2Vm Vm
D2 — ON
Vo, avg –2Vm/π –Vm/π
Equivalent circuit diagram
29. Ans. A, B & D
Sol.: For the first Half circle D 1 is S.C.
Hence

–Vm + Vc1 = 0 Vc1 = Vm

20
www.gradeup.co
For the next negative Half cycle D1 is O.C. 32. Ans B, D
while D2 is S.I. Hence –VC1 – Vm + VC2 = 0 Sol.

Vc2 = 2Vm Consider the figure,

Similarly Vc3 = 2Vm

30. Ans. A, B & D


Sol.: Vi = 3 + 9 sinωt
Maximum input voltage = 12 V
When Diode is in FB, capacitor charges
through Diode up to 12 – 4 = 8 Volt
In Steady state Vc = 8 volt
V o = Vi – V c
Vo = (3 + 9sinωt) – 7
Vo = –4 + 9sinωt Applying KCL at node 1,
DC maximum output voltage = –4 + 9 = 5 15 − 0.7 − Vo Vo
+ ID2 =
V 5k 10k
Minimum output voltage = –4 – 9 = –13 V Vo 14.3 V
ID2 = − + o ...(1)
31. Ans A, B, D 10k 5k 5k
Sol. Applying KCL at node 2,
Case 1:
15 − V2 V
For Vi < 0, then diode D1 is OFF, & diode = ID2 + 2
10k 5k
D2 is ON,
3V2
V0 = 10 – (20k)I ID2 = 1.5  10−3 − ...(2)
10k
10 − 2.5
where I = = 0.25  10−3 A Equating both equations 1 and 2, we get,
30 k
So, V2 – V0 = 0.7
So, V0 = 10 – (20k) x (0.25 mA) = 5 V
Vo V0 14.3 3V2
Therefore, diode D1 will be OFF, up to 5 V + − = 1.5  10−3 −
5k 10k 5k 10k
of input voltage Vi
Therefore, Vo = 6.91 V
Case 2:
For 5 < Vi < 10, the diode D1 is ON, and 3Vo
ID2 = − 2.86  10−3
diode D2 is ON So, 10k
= −0.787  10−3 A = −0.787 mA
So, Vo = Vi
The current through diode from anode to
Vi = 5 V cathode is negative that means the diode
Case 3: D2 is in reverse bias.
If Vi = 9 V, then diode D1 is ON, and diode So, ID2 = 0 A
D2 is ON,
So, option B is correct.
Therefore, Vo = Vi
0k
So, Vo = 9 V Also, Vo = (15 − 0.7)  = 9.53 V
15k
Therefore, option A, B and C are correct. So option D is correct.

21
www.gradeup.co

22
www.gradeup.co
Chapter

2 Voltage Regulators

1. A 10V regulated DC power supply has a 4. The Zener diode shown in the figure has
regulation of 0.002. Find the magnitude of the specifications Vs=15V,Vz=5.2 V and PD
variation in output voltage max = 260 mW. Assume Rz = 0, the
A. 0.02 V B. 0.2 V maximum allowable current iz and the
C. 0.002 V D. None of these minimum value of Rs for which Zener
2. A 24 V, 600 mW Zener diode can be used diode remains in constant reverse
for providing a 24 V stabilized supply to a breakdown region with no longer of failure
variable load. Assume that for proper zener are
action, a minimum of 10 mA, must flow
through the zener. If the input voltage is
32 V. What would be the value of R and
maximum load current?

Vs
A. 50mA, 196kΩ B. 50mA, 196Ω
C. 100mA, 98kΩ D. 100mA, 98Ω

A. 320 Ω, 10 mA B. 400 Ω, 10 mA 5. Consider the following statements:

C. 400 Ω, 15 mA D. 320 Ω, 15 mA 1. A zener diode degrades the input signal

3. Figure shown as electronic voltage at high frequencies due to its transition

regulator, the zener diode may be capacitance.

assumed to require a minimum current of 2. The zener voltage Vz does not vary with

25 mA for satisfactory operation.The value temperature.

of R required for satisfactory voltage 3. Regulation of the zener diode is

regulation of the circuit is adversely affected at the knee current


Iz(min) due to limited power dissipation
capacity.
4. In a simple zener diode regulated
circuit, amplification is not possible.
Which of these statements are correct ?
A. 80Ω B. 160Ω A. 1 and 3 B. 2 and 4
C. 90Ω D. 180Ω C. 1 and 4 D. 1, 2 and 4

23
www.gradeup.co

6. Consider the circuit shown in the figure 8. Calculate the regulated voltages Vo1,
below Vo2 and source current Is in the network
shown.

A. 2.6V, 7.4V and 2mA


The zener diode has a zener breakdown
B. 2.3, 2.3V and 2.5mA
voltage of Vz = 10V and has the maximum
C. 3V, 10V and 2mA
amount of zener current Iz(max) = 20 mA.
D. 3.2V, 6.8V and 4mA
The zener diode turns on when the voltage
9. In the circuit shown below Zener voltage is
of 10V is applied over it with nearly zero
VZ = 5V and β = 100. The value of IC and
Knee current, then the value of load Q

resistance RL for proper regulation should VCEQ are (assume VBE = 0.7V)
be in the range
A. RL > 100Ω
B. 10Ω < RL < 100Ω
C. 250Ω < RL < 500Ω
D. 2500Ω < RL < 500Ω
7. A zener diode is used in the circuit as
shown below which has zener dynamic
resistance of 15 ohms. The zener knee A. 12.47 mA, 4.3V B. 12.47 mA, 5.7V
voltage is 5 V. If the input voltage has a C. 10.43 mA, 5.7V D. 10.43 mA, 4.3V
range from 10 V to 20 V then find the 10. In the following limiter circuit an input
output voltage range if R = 200 Ω voltage Vi = 10 sin(100πf) is applied.
Assume that the diode drop is 0.7 V when
it is forward biased. The Zener break down
voltage is 6.8V. The maximum and
minimum values of the output voltage
respectively are: -

A. 5.348 to 7.2 V
B. 6.04 V to -5.348 V
C. 5.348 V to 6.04 V
D. 6.04 V to 7.2 V

24
www.gradeup.co

A. 7.5v and 0.7v B. 0.7v and –7.5v 14. Consider the circuit shown in the figure.
C. 7.5v and –0.7v D. 1.4v and —3.25v Input voltage varies from 20 V to 30 V and
12. The three terminal linear voltage regulator the load current varies from 0 to 100 mA.
is connected to a 10 Ω load resistor as
shown in the figure: -

Find Rs(min) such that Zener is always ON


and at least a knee current of 10 mA flows
through diode and Calculate the power
If Vin = 10V, the power dissipation in the
rating of the zener.
transistor, (in watt) is_____________
A. 90.9 Ω and 2.2 W
(Assume (VBE)ON = 0.6V
B. 181.81 Ω and 2.2 W
12. The transistor used in the circuit shown
C. 90.9 Ω and 1.1 W
below has β of 30 and ICBO negligible. If
D. 181.81 KΩ and 1.1 W
the forward voltage drop of diode is 0.7V,
MSQs:
then the current through collector will be
15. Consider a Zener Regulator circuit as
shown in the figure below:

The regulator have


(i) Knee current = 5 × 10–3 A
(ii) Maximum allowed power dissipation =
13. A Zenner diode rated 10V, 32 mA can be
300 mW
considered ideal i.e Rz = 0Ω. A source
(iii) Output voltage that is to kept = 6 V
voltage of 50V with RS = 1kΩ is shown in
Then which of the following statement
the following figure. The ratio of maximum
is/are correct with respect to zener
to the minimum value of RL is._____
regulator circuit?
A. Minimum load current is 10 mA
B. Maximum load current is 55 mA
C. Zener diode is working in forward biased
condition
D. Output voltage cannot be maintained at
6V

25
www.gradeup.co

ANSWER

1. A 2. D 3. A 4. B 5. C 6. C 7. C 8. C 9. B 10. C

11. (2.4) 12. D 13. (5) 14. A 15. A,B

SOLUTION

1. (A) V0 = Vz = 10V
Full load voltage VFL = 10V Iz min = 25 mA
Regulation = 0.002 20 − 10
I=
VNL − VFL R
Voltage regulation =
VFL V0 V 10
But IL = = Z =
Variation in output voltage = V NL – VFL 100 100 100

= VFL × voltage regulation IL = 100 mA


= 10 × 0.002 ∴ I = Iz + I L
= 0.02 V I = 125 mA
2. (D) 10 10
R= = = 80
Vz = 24 V, I 125  10−3
Pz = 600 mW 4. (B)
Izmin = 10 mA
PD max 260  10−3
Then, maximum current through resistance iz(max ) = Iz = = = 50mA
VZ 5.2
R
By KVL,
600  10−3 32 − 24
I= = Vs = Riz max + Vz
24 R
Vs − Vz
8  24 Rmin =
Hence, R = = 320 iz max
600  10−3
Minimum current through zener diode is 10 15 − 5.2
Rmin 
mA. 50  10−3

Hence maximum load current = 25 – 10 = = 196 Ω


15 mA 5. (C)
3. (A) A zener diode degrades the input signal at
high frequencies due to its transition
capacitance and for simple zener diode
regulated circuit, amplification is not
possible.
So, only these two statements are correct.

26
www.gradeup.co
I= 15/215
6. (C) Vo= 5 + 15(15/215) = 6.046 V
The minimum value of load resistance can 8. (C)
be calculated when maximum current flows Cut in voltage of Si diode = 0.7 V
through the load. ∴ V01 = 0.7+2.3
Thus, IL(max) = Iin – Iz(min) ∴ V01 = 3v
Iz(min) = 0 knee current nearly V02 = 7 + V01
equal to zero =7+3
IL(max) = Iin ∴ V02 = 10 V
50 − 10 And
Iin = = 40mA
1k Vs − V02
Is =
10 Rs
RL (min) =  103 = 250
40
20 − 10
=
Now, for maximum value of load 5
resistance, we will calculate minimum Is = 2mA
value of current through load. 9. (B)
IL(min) = Iin – Iz(max) Using KVL equation in the input loop
IL(min) = (40 – 20) × 10–3
10
RL (max ) =  103
20
= 500 Ω
7. (C)

VCC = (IC + IB) RC + VZ + VBE


12 = (IC + IB) RC + 5.7
6.3 = 0.5 × 103 × IE

Apply KVL in the loop, 6.3


IE = mA = 12.6mA
0.5
Vo = Vz + I (R+rz) The current in the loop
  
Ic =   IE = 12.47mA
is given by,   + 1
Vi − Vz
I= VCE = VCC – (IB + IC) RC
R + rz
= 12 × (0.5 × 12.6)
(i) If Vi=10 V
= 5.7V
I= 5/(200+15)
VCE = 5.7V
Vo= 5 + 15(5 / 215) = 5.348 V
ICQ = 12.47mA, VCEQ = 5.7V
(ii) If Vi=20 V
10. (C)

27
www.gradeup.co
In positive half cycle, D2 will be off Power dissipated in the transistor

(
If 0  Vi  VD + Vz
1 ) then PT = VCE × IC
PT = 4 × 0.6 = 2.4 W
V o = Vi
12. (D)
( )
If Vi  VD + Vz = 0.7 + 6.8 = 7.5V
1 Zener diode is used as stabilizer.

In negative cycle diode D2 will be ON The circuit is assumed to be as

∴ for Vi < 0
Vo = −VD = 0.7V
2

11. Ans.
Power dissipated in the transistor is
PT = VCE × IC

VB =-11.3
Using KVL equation in loop.
5 -103 (IB) – 0.7-0.7 = 0
103 (IB) = 3.6
3.6
IB = A = 3.6mA
103
IC = βIB = 30 × 3.6 = 108 mA
VC = 0 –ICRC = (–108 × 2.2)= (–237.6)V
We can analyze that the transistor is VB > VC , transistor is in saturation region
operating in active region Note, In saturation IC  I
(VBE)ON = 0.6V
So collector current using KVL in outer
VB – VE = 0.6V
loop.
6.6 – VE = 0.6
0 – (–12) – (VCE)sat = ICRC
VE = 6.6 – 0.6
12 − 0.2
 IC =
VE = 6V 2.2  103
At Emitter by applying KCL equation IC = 5.36mA.
IE = Iβ + IL 13. Ans. 5
VE − VB 6 − 6.6 The circuit with Zener diode regulator is
 I = =
3 3
10 10 drawn below as
VE 6
and IL = = = 0.6A
RL 10

6 − 6.6 6
IE = + = −0.0006 + 0.6 = 0.6A
3 10
10
VCE = VC – VE = 10 – 6 = 4V

28
www.gradeup.co
When Zener diode conduct then voltage Rsmin = 90.9 ohm
across resistance RL is VZ. Power rating of zener diode means
VZ maximum power dissipation of zener diode
IL =
RL which is given by
For RL to be minimum Pz(max) = VzIz(max) ----------------------(1)
IL should be maximum. Iz(max) implies minimum current through load
IS = IL + IZ, resistance i.e., IL(min) = 0 A
When IL max the IZ will be minimum. Vin(max ) − 10V
Iz (max ) = I(max ) =
Vs − Vz 50 − 10 Rs (min)
Is = =
Rs 103
=
(30 − 10) V = 0.22A
IS = 40mA 90.9 ohm

The minimum value of IZ = 0 A From equation 1 we have


(IL)max = IS = 40 mA Pz(max) = 10x0.22 = 2.2W
vz 10 15. Ans A, B
So (RL ) min = =
(IL ) max 40  10−3 A Zener diode is always operating in its
For RL to be maximum reverse Biased condition. As such a simple
IL should be minimum è IZ should be voltage Regulator circuit can be designed
maximum i.e IZ = 32mA using a Zener Diode to maintain a constant
40 mA = (IL)min + 32 mA DC output voltage across the load in spite
è (IL)min = 8mA the variation in the input voltage or change
vz 10 in the load current.
(RL ) max = = = 1250 ohm
(IL ) min 8  10−3 So, option C is wrong.

(RL ) max =
1250
=5
Given I2K = 5 mA
(RL ) min 250 Pd(max) = 300 mW

14. (A) VL = 6 V

Given: Pmax 300  10−3


I2 max = =
IL(min)=0 A, IL(max)=100 mA, Vin(min)= 20 V V2 6
= 50 mA
, Vin(max)= 30 V
Ii = IZ + IL
I = Iz + IL
9−6
Now, Ii = = 0.06 = 60 mA
50
Vin(min) − 10V
Rs(min) = So, IL min = I1 – I2 max = 60 – 50 = 10 mA
I(max )
So, option A is correct
Vin(min) − 10V IL max = Ii – Imin = 60 – 5 = 55 mA (I2 min =
=
Iz (min) + IL (max ) I2k = 5 mA)
20V − 10V So, option B is correct
=
( + 100) mA
10


29
www.gradeup.co

30
www.gradeup.co
Chapter

3
BJT Biasing & Stabilization

1. The common emitter amplifier shown in


the figure is biased using a 1 mA ideal
current source. The approximate base
current value is

A. 1 mA
B. [3.3/(3.3+0.33)] mA
C. [3.3/0.33] mA
D. [3.3(33+3.3)] mA
4. Consider the circuit shown below:

A. 0 Μa B. 10 μA
C. 100 μA D. 1000 μA
2. The stabilization factor (S), for the circuit
shown below is

If the transistor in the circuit is made of


silicon, then the region of operation of the
transistor is
A. forward active region
B. reverse active region
C. saturation region
D. Cut-Off region
101 5. For BJT configuration as shown β = 200
A. 505/6 B.
4
5  10 then base current will be ___ μA
C. 0 D. 1
3. In the circuit of figure, assume that the
transistor has β = 99 and VBE = 0.7 V. The
value of collector current IC of the
transistor is approximately

31
www.gradeup.co

6. For the transistor shown below β = 150.


If IQ = 2mA, (VCE)sat = 0.2V the value of
Vo (in V) is _____

A. 0.987 mA, 3.04 kΩ


B. 1.013 mA, 2.96 kΩ
C. 0.946 mA, 4.18 kΩ
7. In the collector to base bias circuit shown
D. 1.057 mA, 3.96 kΩ
in figure, the value of the RC for
10. The common-emitter current gain of the
VCC = 15V, VCE = 5V, IC = 5 mA,
transistor is β = 75. The voltage VBE in ON
hFE = 100 and VBE = 0.7V.
state is 0.7 V. The value of IE and RC are
(Use VBE(ON) = 0.7V, VCE(sat) = 0.2V for npn
transistor)

A. 1.46 mA, 6.74 kΩ


A. 10 kΩ B. 10.52 kΩ
B. 0.987 mA, 3.04 kΩ
C. 5.68 kΩ D. 1.98 kΩ
C. 1.13 mA, 5.98 kΩ
8. For the circuit shown in figure, all Si
D. None of the above
transistor are identical, VBE = 0.7V, β
11. Consider the circuit shown in figure below.
=125, Then the value of ICN (in mA)
Given that saturation current I = 5 × 10 –
is_____.(take N = 10)
17
Amp, VBE = 800 mV and β = 100
(Vthermal = 26 mV).
The transistor Q1 is operating in

9. The common-emitter current gain of the


transistor is β = 75. The voltage VBE in ON
state is 0.7 V. The value of IC and RC is

32
www.gradeup.co

A. Saturation region
B. Cut of region
C. Reverse active region
D. Forward active region
12. For the circuit shown below, let β = 75.
The Q-point (ICQ, VCEQ) is

A. 10 and 57 B. 20 and 67
C. 30 and 87 D. 40 and 107
15. The circuit in the fig , assume that the
transistor is in active region. It has a large
β and its base-emitter voltage is 0.7 V. The
value of IC is
A. (4.68 mA, 16.46 V)
B. (3.12 mA, 186 V)
C. (3.12 mA, 8.46 V)
D. (4.68 mA, 5.22 V)
13. For the circuit shown in figure given that
IS = 8 × 10–16 A, β = 100 and VBE = 0.8 V.
What is the operating point value?

A. Indeterminate since RC is not given


B. 1 mA
C. 5 mA
D. 10 mA

A. IC = 1.5 mA, VCE = 1.5 V 16. For the circuit shown β = 50, VBE = 0. The

B. IC = 10 mA, VCE = 1.5 V required value of Vi (volts) to get Vo = 5

C. IC = 1 mA, VCE = 1.5 V volts will be ____.

D. IC = 4 mA, VCE = 1.5 V


14. A silicon transistor with β = 45 and
negligible reverse saturation current is
used in the circuit shown. If VCE = 5V, base
current in micro amps and resistance R in
kΩ, respectively, are

33
www.gradeup.co

17. In the silicon BJT circuit shown below A. 12.47 mA, 4.3 V
assume that the emitter area of transistor B. 12.47 mA, 5.7 V
Q1 is half that of transistor Q2. C. 10.43 A, 5.7 V
D. 10.43 A, 4.3 V
20. For the circuit shown below

The value of current IC is approximately


(assume (VBE)ON = 0.7V)
Assume transistors to be identical and VT =
A. 0.5 mA B. 2 mA
25 mV at room temperature, the output
C. 9.3 mA D. 15 mA
voltage Vo in mV is _____. (given I1 = 10
18. For the transistor shown β = 25. Find the
mA I2 = 2mA)
range of V1 Such that
MSQs:
1.0 ≤ VCE ≤ 4.5.
21. For the given power-temperature current
[VBE (ON) = 0.7V]
(power derating curve) for a BJT choose
the correct option(s)?

A. Thermal resistance is 4 c/w


A. 1.86 ≤ V1 ≤ 3.96
B. Slope of the curve gives the value of
B. 2.81 ≤ V1 ≤ 4.46
thermal resistance
C. 1.43 ≤ V1 ≤ 7.96
C. The case to ambience thermal
D. 2.18 ≤ V1 ≤ 3.69
resistance between case to ambience will
19. In the circuit shown below Zener voltage is
be 2°C/W for a junction to case
VZ = 5 V and β = 100. The value of ICQ and
temperature of 1° c/w.
VCEQ are
D. To prevent Heat accumulation inside
BJT, thermal resistance should be smaller.
22. A PNP Si transistor has base width 6 μm
and it is operating at emitter current of 2.2
μA and it is operating at emitter current of
2.2 mA. If Dp = 48 cm2/sec. Then choose
the correct option(s)? (β = 100)

34
www.gradeup.co

A. Resistance of emitter junction = 1.16 to emitter voltage and common emitter


kΩ current gain
B. Resistance of emitter junction = 11.6 C. Stability factor with respect to Reverse
kΩ saturation current has a value lies between
C. α cut-off frequency = 43.35 MHz 1 & (1 + β)
D. α cut-off frequency = 43.35 GHz D. Stability factor for a collector to base
23. For the below power BJT choose the bias circuit depends on collector Resistance
correct option(s) while independent on Base resistance.
25. For the following circuit as shown in figure
below:

A. Required current Rating is 2A


B. Required current Rating is 4A
C. Maximum power dissipation is 24 W
1+
D. Maximum power dissipation is 18 W A. Stability factor is
I
24. Read the following options regarding the 1+ B
IC
stability factor and choose the correct
1+
one(s)? B. Stability factor is
I
1− B
A. For better stability, stability factor must IC
be small
C. If β = 99 then stability factor is 1.43
B. Stability factor is determined with
D. If β = 99 then stability factor is –1.47
respect to Reverse saturation current, Base

ANSWER

1. B 2. A 3. B 4. C 5. (0) 6.(-0.5) 7. D 8. (2.54) 9. A 10.C

11. D 12. D 13. C 14. D 15. D 16.(-2.52) 17. B 18. A 19. B 20.(40.23)

21. C,D 22. B&C 23. A,C 24. A,B &C 25.B,,C

35
www.gradeup.co

SOLUTION

1. (B) Applying KVL in input loop


From the circuit, emitter current IE is 4 – IB(33k) – VBE – 3.3 k × IE = 0
IE = 1mA 4 – IB(33k) – 0.7 – (1 + β) IB × 3.3k = 0
So, the equivalent DC model is 4 − 0.7
IB =
33k + 100  3.3k
So, the collector current is
IC = βIB
3.3  99
= mA
( 3.3  0.33)  100

Therefore, the value of collector current


IC of the transistor is approximately
3.3
= mA
3.3  0.33
Therefore, the base current is obtained as 4. (C)
IE Assume transistor is in active region
IB =
 +1 Apply KVL –10 + 100k IB + 0.7 = 0
1m 9.3
= = 9.9 A IB = = 9.3  10−5 Amp
100 + 1 100k
≃ 10 μA IC = IB = 9.3  10−5  100 = 9.3  10−3 A
2. (A) −10 + 4k  IC + VCE = 0
1+ VCE = 10 – 4 × 103 × 9.3 × 10–3
S=
RC
1 + . VCE = —27.2
R C + RB
∵ VCE = –ve
101 101  501 50501
= = =
1K 601 601 ∴ Transistor is in saturation region.
1 + 100 
501K 5. Ans.
=505/6(nearly)
3. (B)
For dc analysis, we have the circuit

VE – VB = 0.7
VB − VE
IB =
500K

−0.7
= = −1.4A
500K

36
www.gradeup.co
∵ Emitter junction is reverse biased 8. Ans.
So transistor is in cut-off region VE = 0, VB of all transistor is 0.7V
So IB = 0 9 − 0.7
I3K = = 2.767mA
6. Ans. -0.5 V 3K
I3k = IC + IB + IB1 + IB2 + …….IBN
= IC + IB + N.IB
(As transistor are identical,
Therefore, IB = IB1 = ……IBN, IC = IC1 = ….
= ICN)
I3k = IC + (N + 1)IB
= IC + (N + 1) IC/β
 N + 1
VB = 0 V (grounded) = IC 1 +
  
VQ = IE = 2mA
  
Assume BJT is in active region.  IC = I3k  
  + N + 1
    125 
IC =   IE = 2.67  10−3 
1 +    = 2.543mA
 136 
Or IC ≃ IE [for higher β]. ∴ IC = IC1 = …….. = ICN = 2.54 mA.
Using KVL equation 9. (A)
5 = (5 × 103) (2 × 10–3) + VC Given the common emitter current gain, β
5 = 10 + VC = 75
VC = –5V Base-emitter voltage, VBE = 0.7 V
VB = 0 V and VC = –5V Collector voltage, VC = 2 V
VB > VC So BJT is operating under Now, we obtain the required parameters in
saturation region not in active. Then under following steps:
saturation Step 1: For dc analysis, we redraw the
VCB + VBE + (VEC)sat = 0 given circuit as
VC = – (VEC)sat – VBE
VC = (VCE)sat – VBE
VC = 0.2 — 0.7
VC = – 0.5V
7. (D)
VCC − VCE V − VCE
RC = = CC
IB + IC  1
IC 1 + 
 

15 − 5
RC = = 1.98K
−3 101 Step 2: The emitter current (IE) is
5  10 
100 IQ = IE = 1mA

37
www.gradeup.co
Step 3: The collector current (IC) is given
by
 75
IC = I = 1
1 +  E 75 + 1

= 0.986 mA
Step 4: We obtain the collector resistance
(RC) as
Step 2: Assume the transistor in active
5−2 5−2
RC = = = 3.04k region, then determine the collector
IC 9.986
voltage (VC), emitter voltage (VE), base
10. (C)
voltage (VB). By using transistor collector
From the given circuit, we have,
current equation, we have
VEC = 6 V
VBE
(VEC)sat = 0.2 V VT
IC = Ise
Since VEC > (VEC)sat, so the transistor is
800m
operating in active region. Now, we obtain = 5  10−17
26m
the required parameters in following steps:
= 1.153 mA
To determine the emitter current (IE), we
Step 3: So, we obtain the base, collector,
apply KVL in emitter-base section as
and emitter voltages as
12 – IE × 10k – 0.7 = 0
VB = VBE = 0.8 V
VC = VCC – ICRC
= 1.13 mA
= 2 – 1.153 m × 500
The collector current (IC) is given by
= 1.424 V

Ic = I = 1.12mA VE = 0 (Emitter connected to the ground)
1+ E
Step 3: Now, we check either our
Applying the KVL in emitter-collector loop,
assumption (active region) is correct or
we obtain the collector resistance (RC) as
not. The required conditions for operating
12 – 10kIE – VEC – IC × RC + 12 = 0
region of transistor is
12 – 10k × 1.13 – 6 – 1.12 × RC + 12 = 0
VC > VB, VB > VE Active region
24 − 11.3 − 6
RC = = 5.98k VB > VC, VB > VE Saturation region
1.12
11. (D) In above steps, we have determined

For the given circuit, we determine the VB = 0.8 V, VC = 1.424 V, VE = 0

mode of operation (active, saturation, or So, we conclude that

cut-of) of the transistor. VC > VB, VB > VE

Step 1: We redraw the given circuit as i.e. base to emitter is forward biased and
base to collector junction is reverse biased.
Thus, the transistor operates in the
forward active mode.

38
www.gradeup.co
12. (D) Reverse saturation current, IS = 8 × 10–
This is a voltage divider circuit. We obtain 16
A
the operating point in following steps: Now, we obtain the operating point values
Step 1: To determine the operating point, in following steps:
we redraw the given circuit as Step 1: we redraw the given circuit for dc
analysis as

Step 2: The modified circuit parameters Step 2: From the circuit, base current is
are IB = 10 μA
R1 = 25k, R2 = 8kΩ So the collector current is obtained as
RTh = R1 || R2 = 25k || 8k = 6.06 kΩ IC = βIB = 100 × 10μA = 1 mA
 8k  Step 3: Now, we determine the collector to
VTh =   24 = 5.82 V
 25k + 8k  emitter voltage (VCE) by applying KVL in
Step 3: Applying KVL in input loop, we collector-Emitter loop as
determine the base current (IBQ) as VCE = VCC – ICRC
VTh – IBQ × 6.06k – VBE – (β + 1)IBQ × 1k = = 2.5 – (1m)(1k) = 1.5 V
0 14. (D)
IBQ = 62.4 mA Applying KVL in the output loop
Step 4: So, the emitter and collector –VCC+ (IB +IC)RC + VCE + IERE = 0
current is –24 + IE(10K) + 5 + IE(0.27K) = 0
IEQ = (β + 1) IBQ = 4.74 mA IE = 1.85 mA
ICQ = βIBQ = 4.68 mA IE
IB = = 40.2A
Step 4: Therefore, the collector to emitter 1+
voltage (VCEQ) is obtained by applying KVL Applying KVL in the base input loop
in collector-Emitter loop as –VCC + IERC + IBR + VBE + IERE = 0
24 – ICQ × 3k – VCEQ – IE × 1k = 0 –24 + (1.85 × 10–3)(10k) + (40 × 10–6)R
VCEQ = 24 – 4.68 × 3 – 4.74 × 1 = 5.22 V + 0.7 + (1.85 × 10–3)(0.27k) = 0
13. (C) –24 + 18.5 + (40 × 10–6)R + 0.7 + 0.5 =
Given the base to emitter voltage, VBE = 0
0.8 V 24 − 19.7
R = = 107
40  10−6

39
www.gradeup.co
15. (D) VB 9.3
IC = = = 1mA
1 R 9.3k
Given it has large β. So, IB≅ 0A
By using voltage division principle IB = IB + IB (KCL at node A)
1 2
R1  5 
VB = VCC  = (15)   Since (Q2)Area = 2(Q1)Area
R1 + R2  1 + 10 
 D Dp  2
VB = 5V Io = Aq  n +  ni
 LnNA LpND 
VBE = VB – VE = 0.7
Io∝ Area
VE = VB – 0.7 = 4.3 V
IB = 2I1
VE 4.3 2 B1
IC IE = =
RE 430V IC
1
 IB = 3IB1 and IB1 =
IE = 10 mA 1
16. Ans. 1mA
 IB =
700
VBE = VB – VE = 0.7
2  10−3
VB = 0.7 IB2 = 2IB1 =
700
2mA
IC2 = 2I =  715 2mA
2 700
18. (A)
For VCE = 4.5V
KVL in the outer loop

10 − 5
IC = = 1mA
5
IC
IB = = 20A

By applying KCL at base:


−10 − 0.7 Vi − 0.7
+ = IB = 20  10−6
150 20
VCC − VCE
Vi = −2.52V IC =
RC

17. (B) 5 − 4.5


IC = = 0.5 m.A
1
IC 0.5
I = = = 0.02mA.
 25

VBE − ( −5) 0.7 − ( −5)


IR = =
2 RB 100K
2

IR = 0.057mA
2
VB –(–10) = 0.7
IR = I + IR = 0.077mA
VB = –9.3V 1 2

40
www.gradeup.co

V1 = IR .RB + VB Step 4: Since, emitter is connected to


1 1 E
ground, i.e. VE = 0. So, we determine the
So V1 = (0.077)15 + 0.7
collector to emitter voltage (VCEQ) as
So V1 = 1.855V
VC = VE + VBE + VZ
For VCE = 1.0V
= 0 + 0.7 + 5 = 5.7
5 −1
IC = = 4mA Step 5: Thus, the collector to emitter
1
IC 4 voltage is
I = = = 0.16mA
 25 VCE = VC – VE = 5.7 – 0 = 5.7 V

VBE − ( −5) 20. Ans.


IR = = 0.057mA
2 RB As collector and base shorted so it is diode
2
connected BJT mode
IR + I + IR = 0.217mA.
1 2
I1 = I0 e BE1 t 
V /V

V1 = IR .RB + VBE  
1 1

I2 = I0 e BE2 t 
V /V
= (0.217)15 + 0.7
 
= 3.96 V
VBE = VB − VE = VB − 0 = VC
So, range of V1 = 1.86 ≤ V1 ≤ 3.96 1 1 1 1 1

19. (B) similarly VBE = VC


2 2
We obtained the operating point in
lo  e BE2 T 
V /V
following steps. l2 
=  
l1
lo  e BE1 T 
V /V
Step 1: We redraw the given circuit as 
 

l2  VBE2 − VBE1 
= e  / VT
l1  

l 
VBE − VBE1 = VT / n  2 
2
 l1 

l 
V0 = VBE − VBE2 = −VTln  2 
1
 l1 

 2 
Step 2: Applying KVL in collector-base- = −25  10−3 ln   = 40.23mV
 10 
emitter, we determine the emitter current
21. (C, D)
(IEQ) as
‫ •ׇ‬Thermal resistance
12 – 500(IC + IB) – VZ – VBE = 0
12 − 5 − 0.7
1 200 − 50
IE = = 12.6mA = = = 3C / w
500 Slope 50

Step 3: So, the collector current is given • Inverse of slope will give the value of

by thermal Resistance

 QCA = QT – QJC = 3 – 1 = 2°C/w


IE = I = 12.47mA
 +1 E

41
www.gradeup.co
22 (B & C) So it depends on both RB & RC.

VI 26 mV 25. Ans B, C
re = = = 11.6 
IE 2.2 mA For the circuit shown,
𝑤𝛽 2 (6×10−4 )2 Applying KVL in the Base emitter loop and
Base Transit time (τ) = =
2𝐷𝐵 2×48
Let IC = current through collector
360
= n sec IB = Current through Base
98
1 98
 cut-of = =  109 = 43.35
2 2  360
MHz
23. Ans. A & C
VCE =VCC – IC – ICRC = 24 – 8IC

–VCC + (IC + IB) RC + IB + VBE + (IC + IB)RE


=0
Differentiating w.r.t. (IC),
RCE = (VCC – ICRC)IC
For maximum power dissipation  I   I   I 
0 + 1 + B  R C +  B  RB + 1 + B  RE = 0
 IC   IC   IC 
PC V
= 0 → IC = CC
IC 2R C
IB R C + RE 69
24 =− =−
IRating = = 2A IC R c + RB + RE 99
26
Pmax = (VCC – ICRC)IC = (24 – 2 × 6) × 2 1+
Stability factor (S) =
I
Pmax = 24 W 1− B
IC
24. Ans. A, B & C For β = 99,
Sol.; For collector to base bias circuit 1 + 99 100 10
S= = = = 1.43
 69  70 7
1 + 99  
 99 
Therefore, option B and C are correct.



42
www.gradeup.co

43
www.gradeup.co
Chapter

4 Current mirror circuits

1. In the silicon BJT circuit shown below, A. 80 µA, 3V B. 64 µA, 1.72 V


assume that the emitter area of transistor C. 80 µA, 1.4V D. 64 µA, 1.4 V
Q1 is half that of transistor Q2. 4. Consider the emitter follower circuit as
shown in figure. All transistors are
matched with parameters (VBE)ON = 0.7V ,
(VCE)Sat = 0.2V and VA = ∞. Neglecting
base current |Vo max| + |Vo min| is equal to
____ V
[Q2 & Q3 should always be in linear mode].

The value of current I0 is approximately


A. 0.5 Ma B. 2mA
C. 9.3 mA D. 15mA
2. For BJT current mirror as shown β-is very
large. VBE = 0.7, The emitter area of Q1 is
thrice to that of Q2. The value of I(through
Q1)______(mA)

5. Two perfectly matched silicon transistors


are connected as shown in the figure.
Assuming the β of the transistors to be
very high and the forward voltage drop in
3. Find IX and VX if both transistors are diodes to be 0.7V, the value of current I
identical ? is_____

44
www.gradeup.co

A. 0mA B. 3.6mA 8. For matched transistor circuit shown β =


C. 4.3mA D. 5.7mA 100, VBE = 0.7. Then the value of
6. For the devices in the circuit |Vt| = 1V Vx is_____
λ = 0, µncox = 50 µA/V2, L = 1 µm and W =
10 µ m. Find V2 and I2.

A. 4.35 V B. 4.92 V
C. 5.26 V D. 5.56 V
9. If β is very large and thermal voltage V T =
25 mV, then the value of R (in kΩ) is
A. 5V, 1.125 mA
__________
B. 2.5V, 1.125 mA
C. 2.5V, 562.5 µA
D. 5V, 562.5 µA
7. For the circuit shown below current mirror
circuit, find the current Ic2?

10. Consider the circuit shown in figure below,


given that IS1 = 2IS2 = 4 × 10–16 A, β1 =
β2 = 100, and I1 = 1 mA. What is the value
of
VCC − VE − 2VBE VB voltage (in mV)?
A. IC =
2  
2
R 1 + 
  (1 + BR ) 

VCC − VE − 2VBE
B. IC =
2  
1
R 2 + 
 R (1 + B ) 

VCC − VE − 2VBE
C. IC =
2  
1
R 1 + 
  (1 + BR )  11. Choose the correct option(s) regarding
current mirror?
D. None of these

45
www.gradeup.co

A. For proper operation of BJT current 0.8 V, β = 100), then choose the correct
mirrors all transistors must be in saturation option(s)?
region.
B. For proper operation of BJT current
mirror, all transistors must be in active
region
C. For MOSFET current mirror, all MOSFETs
must be in linear region for proper
operation.
D. For MOSFET current mirror, all
MOSFETS must be in saturation region for A. IC10 = 0.86 mA B. IC2 = 0.86 mA
proper operation. C. I = 1.8 mA D. IC3 = 0.86 mA
12. Below figure shows a modified current
mirror circuit (other parameters are VBE =

ANSWER

1. B 2.(10.6) 3.C 4.(9.1) 5. B 6.C 7. C 8. A 9.(38) 10. (817)

11. B,D 12.A,B& D

SOLUTION

1. (B) Since the emitter area of transistor Q 1 is


The given circuit is a current mirror circuit half that of transistor Q2,

in which the output current is a mirror So li/l0=A1/A2


li/l0=1/2
image of the input current if both the
Therefore, l0= 2 mA
transistors are identical.
2. Ans.
∵ VBE = 0.7
VB – VE = 0.7
VB = 0.7 + (–6) = –5.3V
I’ = current through 1.5K resistor
0 − ( −5.3)
I' = = 3.533mA (emitter current
1.5
through Q2)
To calculate Ii 9.3 Ii + 0.7 = 0-(-10) = 10 Emitter area of Q1 = 3 × emitter area of Q2
⇒Ii= 1Ma I = 3 × I’ = 10.6 mA

46
www.gradeup.co
3. (C) Since the two transistors are matches
Initially assume that 1st transistor is IB1 = IB2 = IB
saturation region. VGS is same in both Also, IC = βIB
transistors therefore, ID remains same if IB = I1/ (β+2)
both transistor are identical. Current I is the collector current for Q2.
Ix = ID= 80 μA I = βIB = (β/ β+2) × I
Vx = 3 — Ix (20 kΩ) It β is larger I = I1 = 3.6mA
= 3 – 1.6 = 1.4 V 6. (C)
By substituting Ix and VX, we can conclude V G = VD
that our assumption is true. For Q1, Q2, Q4 (transistor in saturation)
4. Ans. Assume Q3 also in saturation
For Vo to be max, Q1 need to be in 5 = VGS + VGS and ID = ID
1 2 1 2
saturation (for minimum voltage drop
VGS = VGS = 2.5V
1 2
across it)
Q3 and Q4 have the same drain current
(VCE)Q1 = 0.2V
Vo = 5–0.2 = 4.8V VGS = VGS and VGS = VGS = 2.5V
max 3 4 3 4

For Vo min = –5 + 2
    V − VT 
(VCE)Q2 min ……………………………… from current ID = nCox    GS
L  2
mirror property.
2
(VCE)Q2 min = 0.7V  10  (2.5 − 1)
6
ID = 50  10   
1
 1  2
so Vo min = –5 + 0.7 = –4.3V.
So |Vo max| + |Vo min| ID = ID = 562.5A
1 2

4.8 + 4.3 = 9.1V IDS = IDS = ID = I2 [current mirror]


1 3 4
5. (B)
V2 = 2.5 V I2 = 562.5 µA
7. Ans. C.
Iref = Ic1 + IBR
IER
Iref = IC +
1 (1 + BR )
The circuit in the figure shows a current IER = IB1 + IB2

mirror for Q1, because base and collector IER = 2IB1

are shorted ∴IB1 = IB2

VCE = VBE = 0.7v 2Ic


2
Iref = IC +
Applying KVL, we get
1  (1 + BR )

I1 = (5-0.7-0.7)/1 mA = 3.6mA ∴Ic1 = Ic2


I1 = IC1+IB1+ IB2 Iref
IC =
1 2
 (1 + BR )

47
www.gradeup.co

Vcc − 2VBE − ( −VE ) 10. Ans.


Iref =
R For dc analysis, we redraw the given circuit
Vcc − VE − 2VBE as
Iref =
R
Vcc − VE − 2VBE
IC =
2  
2
R 1 + 
  (1 + BR ) 

8. (A)
Emitter is connected to the ground, i.e.
For matched transistor
VE = 0, so
IB1 = IB2, IC1 = IC2
VBE1 = VBE2 = VBE
Since, the collector current in a transistor
is defined as
VBE
VT
IC = Ise

VBE1
VT
So, IC1 = I1 = Is1e

y = IC1 + 2IB1 VBE2


VT
2IC1  2 IC2 = I2 = Is2e
y = IC + 2IB1 = IC + = IC 1 + 
1 1  1 
 Therefore, we get
VBE1
    100 
IC = y   = 1.86   = 1.823mA I1 I e VT
1
 + 2  102  = s1
I2 VBE2
IC =IC2=1.823 mA Is2e VT
1

Vx =8 – 2(1.823) I1
or = 2 (IS1 = 2IS2, VBE1 = VBE2)
= 4.353 V I2

9. Ans. or I1 = 2I2
Vbe1 = Vbe2 + 5μ(R) Again, we have

Vbe1 − Vbe2 IC = βIB


R=
5 So, I1 = β1 IB1, I2= β2 IB2,
Since β1 = β2, so we obtain
I  10m 
VT n  C1  25m n   IB
I I1
=  C2  =  5  = 38k = 1 =2
5 5 I2 IB
2

Hint: IC = IS e Vbe/VT
Therefore, the base currents are obtained
⇒ Vbe = VT ln (IC/IS) as
Vbe1 = VT ln (IC1/IS) I 1  10−3
IB1 = = = 10A
Vbe2 = VT ln (IC2/IS) 1 100

I  IB1
 Vbe1 = Vbe2 = VT ln  C1  and IB2 = = 5A
2
 IC2 

48
www.gradeup.co
Also, we obtain the base-emitter voltage I = IC1 + IB1 + IC2 + IB2 + IB3
as  1  1  IC
I = IC1 1 +  + IC2 1 +  + 3
 I      
VBE = VBE1 = VBE2 = VT ln  1 
 IS1  Due to mirror action
 1  10−3  IC1 = IC2 = IC3
= 26  10−3 ln   = 742mV
 4  10−16 
   3
I = IC1 2 + 
Applying KVL in base emitter loop, we get  
VB – (IB1 + IB2)5k – VBE = 0 1.74
IC1 = = 0.86 mA
Or VB = 15 × 10–6 × 5 × 10–13 + 0.742 = 2 + 0.03
0.817 V = 817 mV IC3 (1 + )
IE3 = = IC3
11. (B & D)  
12. (A, B & D) 101
IE3 = 0.86   0.86 mA
For the gives circuit 100

9.08 8.2
I= = = 1.74 mA
4.7k 4.7k



49
www.gradeup.co

50
www.gradeup.co
Chapter

5 BJT Amplifiers

1. The voltage gain of the amplifier is RL


A. B. –gm RL
1
+1
gm

−R C
C. D. gm RL
1
+1
gm

4. The purpose of emitter bypass capacitor in


a CE BJT amplifier is to
A. increase the mid band voltage gain of
the amplifier
−R C −RE
A. B. B. provide a stable biasing for the amplifier
RE RC
C. place the Q-point of the transistor in
R C.hfe R C .hfe
C. D. active region
R1 || R2 R1 + R 2
D. prevent saturation of the amplifier
2. An amplifier with input resistance of 15 kΩ
−VC
is driven by current source of 2μA and 5. Find the voltage gain A V = of the
Vs
source resistance is 150 kΩ, has a short
given amplifier circuit.
circuit output current of 12 mA and open
(Assume VT = 25 mV)
circuit voltage of 24V. When driving a load
of 2 kΩ load, the voltage gain in dB is
______
3. For the BJT configuration shown below
V  i
find  o  if gm = c
 Vs  Vbe

A. 20 B. 75
C. 95 D. 150
6. The transistor in the circuit of figure is
biased at a dc collector current of 0.5 mA.
(Assume VT= 25 mV)

51
www.gradeup.co

9. The voltage gain (V0/V1) of the amplifier


circuit is ___ (VT = 26mV)?

The voltage gain is


A. –150 B. 150 10. In the ac equivalent circuit shown, the two
C. 100 D. –100 BJTs are biased in active region and have
7. For the following given circuit, find the identical parameters with β >> 1. The
small signal voltage gain for β = 80 & VA = open circuit small signal voltage gain is
∞ and VT = 26 mV. approximately___________

A. – 61.7 B. – 47.4
A. 1 B. -1
C. – 144 D. – 166.23
C. 0 D. 2
8. For ‘Si’ transistor circuit shown α =0.98,
11. Consider the circuit shown below. The
VBE = 0.7V, VT =25mV. Then the value of
transistor parameters are β = 120 and VA
Ri (in Ω) as shown in figure
= ∞ (Assume VBE = 0.7 V and VT = 25.9
is____________.
mV)
(Rounded upto two decimal point)

52
www.gradeup.co

The hybrid-π parameter values of gm,


rπ and ro are :
A. 24 mA/V, ∞, 5 kΩ
B. 24 mA, 5 kΩ, ∞
C. 48 mA/V, 10 kΩ, 18 kΩ
D. 48 mA/V, 18.4 kΩ, 10 kΩ
12. If the parameters of the transistor are β =
100 VT = 25 mV & IC = 0.5 mA 1) Large output resistance

then find the input resistance? 2) High current gain

(R1=R2=10kΩ) 3) Voltage gain AV = –gmRC


4) No early effect
5) No miller effect
A. 1, 3 and 5 B. 1, 4 and 5
C. 2, 3 and 5 D. 1, 3, 4 and 5
15. Consider the transistor amplifier circuit
shown below. The transistor parameters
are given as β = 100, VBE = 0.7, VA = ∞.
A. 5KΩ B. 10kΩ io
Find the current gain ? (VA = 26mV)
C. 2.5kΩ D. 7.5kΩ i1

13. Consider an amplifier circuit shown in


figure. If the transistors θ1 and
θ2 parameters gm1, rπ1 and gm2,
rπ2 respectively then voltage gain |Av| is

16. The current ib through the base of a silicon


npn transistor is 1 + 0.1 cos(10000 πt)
mA. At 300 K, the rπ in the small signal
model of the transistor is

gm2r2 gm1r1
A. B.
1 + gm2r2 1 + gm2r2

gm1r2 gm1r1
C. D.
1 + gm2r2 gm2r2

14. Which of the following statements is/are A. 250 Ω B. 27.5 Ω

correct about this amplifier ? C. 25 Ω D. 22.5 Ω

53
www.gradeup.co

Consider the circuit shown below. The If the lowers out off frequency is 20 Hz ,
transistor parameters are β = 120 and then the value of capacitror CC is ----μF
VA = ∞. (Vthermal = 0.0259 V) A. 0.53 μf B. 0.62 μF

C. 0.78 μF D. 0.67 μF

MSQs:

21. Consider the following circuit. The

transistor parameters are β = 120 and VA

=∞

17. Find the value of rπ (in k ohm)___


18 The small signal voltage gain AV = V0/VS is
19. The transistor in the amplifiers circuit
shown in figure is biased at IC = 1mA.
(Use VT = KT/q = 26mV, β0 = 200)

Choose the correct option(s)?

A. Value of Hybrid parameter rπ = 5 kΩ

B. Value of Hybrid parameter gm = 24

mv/A

C. Value of Hybrid parameter ro = ∞


What is the required value of CE (in mF) for
D. Value of voltage gain of small signal = –
the circuit to have a lower cut off
1.88 V
frequency of 10 Hz?
22. For the below high frequency π model,
20. Consider an amplifier circuit shown in
figure below having Vthermal = 26 mV. Read all the options and choose the correct

(provide that VBE = 0.6V) option(s)?

If the value of Thermal voltage is 26 mV

and collection current is 1 mA. Base

current is 10 μA.

54
www.gradeup.co

A. β cut off frequency of the BJT will be 24. Consider h-parameters of shown common
10.2 Hz emitter amplifier shown as
B. Unit gain frequency of the BJT will be hie = 1500 Ω, hfe = 100, hoc = 0, hre = 0
10.2 Hz
C. β cut-off frequency of the BJT will be
1.02 kHz
D. Unity gain frequency of the BJT will be
1.02 kHz
23. For the below amplifier having hfe = 120 hi
= 2.2 kΩ.
Choose the correct option(s)? For the above configuration, which of the
following statement is/are correct?
A. Input impedance is 8.3 kΩ
B. Output impedance is 2.8 kΩ
C. At mid band frequency voltage gain is –
1.4
D. Input characteristic is shown as

A. Medium frequency voltage gain will be –


163.64
B. Overall input Resistance will be (Seen
by Vi) 2.18 kΩ
C. Lower 3 dB frequency will be 6.63 Hz
D. Ro (Output Resistance) will be 3 kΩ

ANSWER

1. A 2. (52.86) 3. D 4. A 5. D 6. D 7. B 8. (17.65) 9. (-3) 10. B

11. B 12. C 13.C 14. D 15. (0.98) 16. C 17. (5) 18. (-1.88) 19. (0.2652) 20. A

21. A,C&D 22. A,D 23. A,B 24. A,B & C

55
www.gradeup.co

SOLUTION

1. (A) 3. (D)
The small signal equivalent circuit of given
transistor is

Vo = –ic RL (By KVL in loop 1)


Vo =—hfe ib Rc ……(i)
Vs = – Vbe (by KVL in loop 2)
Apply KVL for i/p loop
Vo −i R
Vi = (hie + (1 + hfe) RE)ib……..(2) = c L = gmRL
Vs −Vbe
Vo −hfeibR c
Ao = = 4. (A)
Vi (hie + (1 + hfe ) RE ) ib The purpose of emitter bypass capacitor in
−RC a CE BJT amplifier is to increase the mid-
A
 hie 
 + RE  band voltage gain of the amplifier.
h
 fe 
5. (D)
(1 + hfe = hfe) Using DC analysis,
hie
As RE 
hfe

−R C
A =
RE

2. Ans.
24
Ro of amplifier = = 2k
12mA

It is clear from the figure


Vin = 2 × 10–6 (150k || 15k)
IE = 0.5 mA
= 0.02727
IB = 0
2
Vo = 24  = 12V IC = IE = 0.5 mA
2+2

 12  IC 0.5mA
AV (dB) = 20 log10  gm = =
 = 52.86dB VT 25mV
 0.02727 

56
www.gradeup.co
Small signal equivalent circuit So, we get the transistor parameters as

IC = IB = 0.743mA
VT 80  0.026
r =  = = 2.79k
ICQ 0.743
VA
ro = =
ICQ

Now, the small signal model of given circuit

is
Vπ = –VS
Output voltage VC is given as
VC = –gmVπRC
⇒ –gmVπ × 7.5 K = gmVS × 7.5 × 103
VC
AV = = 20  7.5  103  10−3 = 150
VS
From the circuit, the output voltage is
6. (D)
Vo = (βIb (3.7 k || 10k))
AV = -gm(RC // RL)
Applying KVL in the input loop,
= –gm[10k // 10k]

−0.5mA IC 0.5mA Vs = —(2k + 2.79k)Ib


=  5K ( gm = = )
25mV VT 25mV So, voltage gain is obtained as
= –100
Vo 
AV = =−  (3.71k || 10k )
7. (B) Vs 2k + 2.79k
For the given circuit, the dc model is 80  2.85k
=− = −47.4
shown in figure 4.79k

8. Ans.

 0.98
Given α = 0.98 ,  = = = 49
1 −  0.02

To calculate re apply DC analysis is

VB = 0, VE = -0.7 V

−0.7 + 9
IE =
6k

=1.383mA

VT
re =
Applying KVL in emitter-base loop, IE
9 — 11k × IE — VEB — 2k × IB = 0 25
=
9 − 0.7 1.383
Or IB = = 9.29A
81  11k + 2k
=18.07Ω

57
www.gradeup.co
10. (B)
Since VBE (Emitter-Base Voltage) = 0.7V

AC equivalent circuit to

VBE = 0.7V
Vin = −VE = 0.7V
Vin = 0.7V
Vout −0.7
So, = = −1
I'e = -(1+β) Ib Vin 0.7
Ve reIb  11. (B)
Zi = = = r
Ie (1 +  ) Ib (1 +  ) e Using dc analysis
49
Zi =  18.07 = 17.71
50

Ri=6k||zi=6000||17.71
= 17.65Ω
9. Ans.
Apply DC analysis
IB = (2.7 -0.7)/(100x103) KVL equation in the input side loop
= 20 μA VBB – VBE = IBRB

IE = (1+β)IB 1.3
IB =
250k
= 2mA
IB = 5.2 µA
re= VT / IE
IC = βIB = 120 × 5.2 µA = 0.642 mA
= 26/2
IC 0.642mA
= 13 Ω gm = = = 24mA / V
VT 0.0259V
Then the gain AV = (-β/1+β) RC/re VT  120
r = = =
= -228.46 IC gm 24mA / V

Zi = (1+β)re = 1300 ohm rπ = 5kΩ

A = V0/ VI = AV (Zi/Zi+RB ) VA = ∞ (given)

= -228.46 (1300 /1300 +100 x103) So, ro = VA /Ic


=∞
A= -3

58
www.gradeup.co
12. (C) = gm1re2 (1)
VT  
r =  now,re = &=
IC gm  +1
25 
= 100  & r =
gm
(2 )
0.5
r = 5k r 
so, =
re 
Drawing AC equivalent ckt,
 
r = re = re
   
  + 1
 
rπ = (β + 1) re
 r 
  re =  
∴ Vin = Ii × (R1||R2||rπ)   + 1

Vin & β = gm rπ (from equation (ii))


 Ri = = R 1 || R2 || r
Ii r
re =
= (10kΩ || 10kΩ) || 5kΩ 1 + gmr

Ri = 2.5kΩ Putting in eqn (1), then


13. (C) gm1r2
Av =
draw the small signal equivalent model of 1 + gm2r2

the circuit. 14. (D)


It is a wide band amplifier i.e. cascade
amplifier.
It has the following properties.
Vo
1. A V = = −gmR C
Vin

2. Wider bandwidth (No miller effect)


Since, the above current source is short 3. Large output resistance (No early effect)
circuited, then can be remove 15. Ans.
So, equivalent circuit is For the DC equivalent circuit

10 − 0.7
IE = = 0.93mA
So, vo = – gm1vin × re2 10k
IC = α IE = 0.921 mA
Vo
Av = = −gm1re2
Vin

59
www.gradeup.co

IC 17. Ans.
gm = = 0.03542A / v
VT For the given circuit, first draw the dc

 equivalent circuit, and then obtain the


re = = 27.94
gm hybrid-π- parameters.
Now, draw the ac equivalent circuit. Step 1: For dc analysis, replace all the AC
sources by their internal resistance.

10k
i1 = i1
10k + 27.94
I1’ = 0.9972 i1
vbe = – 27.94 i1’ Step 2: Now, apply KVL in input loop to
= – 27.94 × 0.9972 i1 find base current (IB)
= – 27.86 i1 2 – IB(250k) – VBE = 0
io = – gmvbe
2 − 0.7
= – 0.3542 × – 27.86 i1 IB = = 5.20 A
250k
io = 0.986 i1
Step 3: For the given value of β, the
i0 collector current is given by
= 0.98
i1 IC = βIB = 120 × 5.20μA

16. (C) = 0.624 mA

We know that Step 4: Using the above results, determine

rx = (  + 1)re the hybrid-π parameters gm, rπ, and r0 as


IC 0.624m
V
rx = (  + 1) T gm = = = 24mA / V
Ie Vt 0.0259
VT  120
rx = (  + 1) r =  re = = = 5kΩ
(  + 1)I b gm 24m
VT 18. Ans.
rx =
Ib For ac analysis, we modify the given circuit
Where lb is d.c. current through base so as
Ib = 1mA a. All the capacitors are short circuit
VT = 25mV at room temperature b. All the dc supplies are removed.

25 10−3 So, we get the hybrid-π model of the


So, rx = = 25Ω
110−3 circuit as

60
www.gradeup.co
Putting the value of Ib in equ (i), we get
1 1 
I = + +
RE RB + rz RB + rz
1 (  + 1)
= +
Apply KVL in output loop, RE ( RB + r )
V0 = –gmVπRC
R eq =
( RB + r ) RE
By voltage divider rule in input loop,
( RB + r ) + RE ( + 1)
r
V =
r + R B
Vs ( RB + r )
 +1
Substituting the value Vπ, =  RE
( RB + r ) + R
V0 = −g m R c
r
Vs  +1 E

r + R s
 R +r 
Thus, the small signal voltage gain is = RE  B  
  +1 
V0 r
AV = = − g m RC 26mV
Vs rz + RB r =  re = 200 
1mA
5k = 5200 = 5.2kΩ
= −(24m)(4k ) = −1.88
5k + 250k
 30.2k 
19. Ans.
R eq = 100  
 201 
As given circuit consist of both dependent
= 100 || 150.25
and independent sources so find equivalent = 60.06 Ω
resistance across RE, we apply 1V source 1
f0 =
across RE by short circuit existing voltage 2 Req CE
source as
1
CE = = 0.2652mF
2  60.04 10
20. (A)

Apply KCL of point Ix


1 1 Thevenin equivalent of the given circuit,
−I x + + + ( − Ib  ) = 0
RE RB + r
1 1
Ix = + − ( I b  ) …………(i)
RE ( RB + rz )

−1
Ib =
RB + r

61
www.gradeup.co

20k (12V ) 21. (A,C & D)


VTh = = 1.2V
200k 2 − 0.7
RTh = 20k 180k = 18k IBQ = = 5.2 A
250k
Step 1: KVL for input loop ICQ = βIBQ = 120 × 5.2 = 0.694 mA
VTh − VBE 1.2V − 0.6v
IB = = = 6.06 A 𝑔𝑚 =
𝐼𝑆𝑄
=
0.624
= 24 𝑚𝐴/𝑉
RTh + (1 +  ) RE 18 + 81k 𝑉𝑇 0.0259

⇒ IE = (1+β) IB = 0.49 mA 𝛽 120


𝑟𝜋 = =  𝑘𝛺
𝑔𝑚 24
I E 0.49mA
⇒ gm = = = 0.0188
VT 26mA rπ = 5 kΩ

 80 VA = ∞ ⇒ rD = ∞
 hie = r = = = 4.255kΩ
gm 0.0188 Small signal equivalent

Now, the lower cutoff frequency is defined


as
1
fL = ..............(i)
2 R eq CC
Where Req is the equivalent resistance Vo r
= −9m(4 k)
across capacitance cc. Vi (r + RB )
To determine the value of Req, use redraw
Vo 5
the circuit as = −24  4  = −1.88
Vi (5 + 250)

22. (A & D)

β cut off frequency fs = fI

fT = unity gain frequency

IC 1 mA 1
Im = = =
VT 26 mA 26
From the equation we obtain
Im 1
Req = RB || (rϖ+(1+β) RE)
fT = =
2(c + c ) 26  2  6  10−6
(∴ rϖ = 4.25kΩ)
= 18k||(4.25k + 81×1k) 106
fT = = 1.02 kHz
312
= 14.88 kΩ
Substituting it in equ (i), we get fT f 1.02 kHz
f = = T =
1  IC 100
fL =
2R eqCC IB

1 f = 10.2 Hz
CC =
2  3.14  14.88k  20
= 0.53 μ F

62
www.gradeup.co
23. (A & B) Zin = [hie + (1 + hfe)RE] || (10k || 100k) =
[1.5 + (101) × 2] || (10 || 100)
= 8.35 kΩ
So, option A is correct.
Output impedance,
Z0 = RL || RC = (10 || 4)
ℎ𝑓𝑒 (6||6) 𝑘𝛺 120 × 3 10  4
𝐺𝑎𝑖𝑛 = − =− = = 2.85 k
ℎ𝑖𝑐 2.2 14
Gain = –163.64 So, option B is correct.
Ri = RB || hie = (250 || 2.2) kΩ Voltage Gain
Ri = 2.18 kΩ Vo −hfe ib (Z0 )
=+ =
1 1 Vi ib (hie + (1 + hfe )RE )
fL = =
2(R L + R L )C 2  (6 + 6)  0.5  10−3 2.85 k
= −100  = −1.4
000 1.5 + (202) k
fL = = 26.52 Hz
12
So, option C is also correct.
Ro = 6 kΩ
Input characteristics of amplifier is input
24. (A, B, C)
current Vs input voltage keeping output
The small signal equivalent model of circuit
voltage constant.
is
Input current = IB; Input voltage = VBE
Output voltage = VCE
So option D is incorrect.

So input impedance



63
www.gradeup.co

64
www.gradeup.co
Chapter
JFET Biasing & Amplifiers
6 (Only ESE)

1. Which of the following is wrong in case of 6. Statement (I): JFET is operated in


JFET depletion mode only.
A. It has large input impedance Statement (II): The input resistance of a
B. value of current through gate terminal MOSFET is several orders of magnitude
is zero greater than that of a JFET
C. JEFT is always operated in saturation A. Both Statement (I) and Statement (II)
region for amplification are individually true, and Statement
D. It is used as a wideband amplifier (II) is the correct explanation of
2. When the controlling voltage of an n- Statement (I).
channel JFET is kept at 0 V, and a voltage
B. Both Statement (I) and Statement (II)
greater than the pinch-off voltage is
are individually true but Statement (II)
applied between the drain and the source
is not the correct explanation of
terminal, the JFET:
Statement (I).
A. Gets into the cut-off region
C. Statement (I) is true but Statement
B. Has a constant output voltage
(II) is false.
C. Acts as a current source
D. Statement (I) is false but Statement
D. Acts as a variable resistor
(II) is true.
3. The main drawback of a JFET is its:
7. Which of the following symbol correctly
A. high input impedance
represents a p-channel JFET?
B. low input impedance
C. higher noise
A. B.
D. lower gain
4. Compared to BJT, the JFET has a much
higher
A. voltage gain B. Input resistance
C. supply voltage D. Current
5. The transconductance of a JFET is C. D.
computed at constant VDS by:
A. Ratio of change in Id to change of Vgs
B. Ratio of change in Vgs to change of Id
C. Product of change in Vgs to change of Id
D. Ratio of change in Vds to change of Id

65
www.gradeup.co

8. An n-channel JFET having VP = –4V and A. 2.5 B. 5.3


IDSS = 10mA is used in the circuit of figure. C. 6.4 D. 8.0
The parameter values are VDD = 18V. RS = 10. Determine the trans-conductance for a
2kΩ, RD = 2kΩ R1 = 450 kΩ , R2 = 90 kΩ . JFET at VGS = –2V,
Determine ID and VDS. if gm0 = 8 mS and ID = IDSS/6
A. 2.6 Ms B. 3.2 mS
C. 4 mS D. 4.8 mS
11. Determine the value of the trans-
conductance for a JFET at VGS = 0V,
If gm = 2 mS
at VGS = –4V.
Given, VP = –6V
A. 2 mS B. 4 mS
A. 4.9 mA, 1.6V B. 4.9 mA, 8V C. 6 mS D. 8 mS
C. 2.5 mA, 8V D. 2.5 mA, 1.6V 12. An n- channel JFET having pinch off
9. Determine the maximum drain current in voltage Vp = -2V, is biased such that, the
mA for the JFET in the given network, if
drain current is 2 mA and saturation
VGS = –3V and VP = –8V
current IDSS = 8 mA. The transconductance
of JFET is
A. 4 mA/V
B. -2 mA/V
C. -4 mA/V
D. 1.414 mA/V

ANSWER

1. D 2. C 3. D 4. B 5. A 6. B 7. D 8. C 9. C 10. B

11. C 12. A

66
www.gradeup.co

SOLUTION

1. Ans. D. 3. Ans. D.
→ FET has zero input current i.e IG = 0 JFET has relatively low gain-bandwidth
Vi product compared to conventional
now Ri = =
IG
transistors. JFET’s theoretically are ideal
So, it has large input impedance. voltage amplifiers with high input
It is used as resistance in linear region &
resistance and low output resistance. But it
used as amplifier in saturation region.
is seldom used in amplifier circuits due to
As it has small gain – bandwidth product, it
its low gain bandwidth product compared
cannot be used as wideband amplifier
to Bipolar Junction Transistors.
2. Ans. C.
4. Ans. B.
The transfer characteristics for an n-
Compare to BJT, JFET has very small input
channel JFET when the controlling voltage,
VGS = 0V is shown below. current approximately zero so it has very
large input resistance.
5. Ans. A.
Transconductance is the electrical
characteristic that relates the output
current to the input voltage. For a JFET,
the transconductance is computed as the
ratio of the change in drain current to the
It can be seen from the characteristics that
when the voltage between the drain and change is gate-to-source voltage.

source (VDS) becomes greater than the ID


gm =
VGS
pinch-off voltage (Vp),the drain current
remains essentially the same, as the It expresses the performance of a JFET.
region between the two depletion regions Larger the trans-conductance, greater the
will increase in length. gain it can deliver.
The drain current ID is fixed at the value ID 6. Ans. B.
= IDSS, where IDSS is the maximum JFET is operated only in depletion mode
draincurrent. while depletion type MOSFET can be
Hence, the JFET acts as a current source
operated in both depletion and
as shown below, for VGS = 0V, VDS > Vp
enhancement modes.
7. Ans. D.
A p-channel JFET with applied voltages is
shown below.

67
www.gradeup.co

For IDS = 2.5 mA


VGS = 3 — 2 × 2.5 = —2V
VDS = 18 × 4 × 2.5 = 8V
Since VDS > (VGS – VP)
So, ID = 2.5 mA
VDS = 8V
9. Ans. C.
Considering input loop,
Since, IG = 0A and IS = ID,
As it can be seen that when the p-n VGS = –IDRS
junction is forward-biased, the gate current VGS 3
 ID = − = = 2.5mA
RS 1.2k
IG is flowing out of the p-channel device.
Similarly, the drain current ID will also flow Since,
2
out of the device.  V 
ID = IDSS 1 − GS 
Hence, a p-channel JFET is correctly  VP 

represented by the symbol given below. Substituting values,


2
 −3 
 2.5mA = IDSS 1 −
 −8 
 25 
 2.5mA = IDSS  
 64 
64
 IDSS =  2.5mA = 6.4mA
25
10. Ans. B.
The drain current for a JFET is given as,
2
 V 
8. Ans. C. ID = IDSS 1 − GS 
 VP 
90
VGG =  18 = 3V  V  ID
540  1 − GS  =
VGS = VGG − IDR S  VGS = 3 − 2ID  V  I
 p  DSS

 V 
2

ID = IDSS 1 − GS   ID = 10 1 +
(3 − 2ID )  Since, the trans-conductance for a JFET is

 VP   4  given as,
2

IDS = 10 1 +
(3 − 2ID )   V 
gm = gm0 1 − GS 
 VP 
 4  
ID
On solving we get  gm = gm0
IDSS
IDS = 4.9 mA, 2.5 mA
Substituting values,
For IDS = 4.9 mA
IDSS / 6
VGS = 3 — 2 × 4.9 = —6.8V  gm = 8 mS 
IDSS
VDS = 18 – (4) × 4.9 = —1.6V
1
 gm = 8 mS  = 8mS  0.4 = 3.2mS
6

68
www.gradeup.co

11. Ans. C.  −4  1


 2 mS = gm0 1 −  = gm0  
Trans-conductance for a JFET is given as,  −6  3
 gm0 = 2  3 = 6mS
 V 
gm = gm0 1 − GS 
 Vp  12. Ans. A.
2
where gm0 is the trans-conductance at VGS gm = − ID  IDSS = 16  10−6
Vp
= 0V
= 4mA / V
Hence,



69
www.gradeup.co

70
www.gradeup.co
Chapter

7 MOSFET Biasing
B & Amplifiers

1. For the following given circuit with

following parameters, gm = 1mA/V, ro = 50

kΩ. What will be the voltage gain

V0
AV = ?
Vs

A. 1.231 B. 9.679

C. 1.034 D. 0.517

4. Compare the node voltages of the FETs if

 
kn   = 0.4 mA / V2 , Vt = 1V and λ = 0
L 

2. For the given transistor circuit having

following parameter, gm = 2mS, ro = 100K,

find the output impedance?

A. V1 > V2

B. V1 < V2

C. V1 = V2

D. Max (V1, V2) = –1.7V

A. 100 kΩ B. 1.33 kΩ 5. In the circuit shown below, all transistor is

C. 0.498k D. 50k n-channel enhancement mode. MOSFETS

 Vo  they are identical and are biased to


3. Find voltage gain   of the circuit
 Vin  operate in saturation mode. Ignoring

shown, if gm = 2 × 10–3 S and rds = 30 kΩ. channel length modulation, the output

voltage Vout is ………….

71
www.gradeup.co

8. Consider the self-bias circuit shown below.


The parameter of MOSFET follows Vt =
0.6V

6. Consider a circuit shown if in figure below.


The parameter of NMOS is given

kn = 60A / V2 ,
W 220
= ,
L 6
VTN = +1V For the value of drain current ID = 0.8 mA
and a transconductance gm = 10 ms, the
value, of resistor RD is …. kΩ (rounded up
to three decimal place)
9. Consider the amplifier circuit shown in
figure below:

If voltage drop across RD and RS is equal.


So the value of RD is in kΩ is ……...
(rounded up to two decimal value) If effect of channel length modulation is
7. Consider a circuit shown in figure, If IREF = negligible, then input resistance R in of
2mA, VDD = 3V, Vth, = 0.4V, Kn' = 400 equivalent circuit of amplifier?
micro Ampere/volt and R = 1kΩ
2
A. gm
1
B.
gm

1
C. (RL RD ) 
gm

1
D. R si
gm

10. Consider the NMOS transistor as shown in

figure. The MOSFET has parameters


To generate, output current. I0 = 10mA,
nCox W
ratio of (W/L)2 is ………. (rounded up to two = 0.5 mA/V2 , VT = 2V and λ = 0.
2L
decimal value)

72
www.gradeup.co

The transistor is used to amplify the small

signal Vin as shown in the figure. If the

value of signal Vin = 3sin(wt) mV, then the

value of output signal V0(t) is equal

to______

13. Find the labelled node voltage if Vt = 1 V


 
and kn   = 2mA / V2 ( = 0)
L 

A. –30 sin(ωt) V B. –15 cos(ωt) V

C. –15 cos(ωt) mV D. –30sin (ωt) mV

11. What will be the maximum allowable peak

input signal for the given common source

amplifier. Given Vt = 1.5V

 
Kn   = 0.25mA / V2 , VA = 50V
L  A. V1 = 5.56, V2 = 1.55
B. V1 = 2.44, V2 = 1.55
C. V1 = 2.44, V2 = –2.56
D. V1 = 5.56, V2 = –2.56
14. The NMOS and PMOS transistor are
matched with
   
kn   = kp   = 1mA / V2 ; Vtn = −Vtp = 1V ,
L  L 
A. 1.5V B. 0.34 V Assume λ = 0 for both devices.
C. 4.3 V D. 4.4 V Calculate the output voltage Vo (in Volt) for

12. For the following given circuit with Vin = 2.5 V

following parameters, gm = 1mA/V ,ro = 50

kΩ. What will be the voltage gain

Vo
AV = ?
VS

73
www.gradeup.co

15. Consider the following Circuit shown below

Current flowing through R2 equals to one


Transistor parameters are given as:
tenth (1/10) of ID (Drain current). For the
λ = 0, Vth = 0.4V
value of Drain current ID = 0.5mA, the
µncox = 200 µA/V2.
value of resistors R1 and R2 are
RD = 1.4 kΩ to maintain the M1 in
respectively?
saturation.
A. 5.32k,10.65k
If the voltage gain AV = 5, then parameter
W
B. 1.068k,14.932k
 L  is
  C. 10.68k,3.32k
A. 63.80 B. 31.90 D. 3.32k,10.68k
C. 22.56 D. 15.95 18. Consider the common source circuit shown
16. Which one of the following statements below.
is/are true with regards to channel Length
modulation in a MOSFET device.
i) It is similar to base width modulation
in BJTS.
ii) The Pinch-off point relocates with
respect to applied drain voltage
iii) Drain voltage influences the current-
voltage characteristics of a MOSFET in
If the Transistor parameter are given as λ
saturation.
= 0, VTH = 0.6V, μn CoX = 120 μA/V2. If
A. only I
resistance RD= 800 Ω and voltage gain.
B. only ii
AV = 8.0, than parameter. (W/L) is
C. only (i) and (ii)
A. 410.8 B. 520.8
D. only (i), (ii) and (iii)
C. 350.8 D. 702.2
17. In the circuit shown below, the MOSFET
19. Consider the circuit shown in figure below.
parameters are as follows:
If the Lower corner Frequency due to
VTN = 0.4V
coupling capacitor CC is fL = 25 HZ, then
μnCox = 200μA/V2
the value of coupling capacitor CC is ____
W
= 278
L

74
www.gradeup.co

μF (Answer should be correct up to three 22. A transistor amplifier is fed with a signal
decimal value). source having an open circuit voltage
Vsig of 10 mv and an internal resistance
Rsig of 100 kΩ. The input voltage Vin and
the output voltage V0 are measured both
without and with load resistance RL = 10
kΩ connected to amplifier output. The
measured results are as follows:

20. The input impedance (Zi) of the circuit


shown in figure is _____ kΩ. Assume VTN =
0.5V, Kn = 2mA/V2 λ = 0

If transistor is modeled as shown in fig 2,


21. Consider the circuit shown in the figure then the value of output resistance R0 of
below: the amplifier is ____ kΩ (rounded up to
two decimal value)
23. For the circuit shown below transistor
parameter are VT = 1V Kn = 0.75
mA/V and λ =0, If The transistor is in
2

saturation
The DC biasing of the circuit is such that
the circuit is always in saturation region for
all values of small signal input source Ii.
The frequency of input source Ii is such
that all the coupling capacitors acts as
short circuit. If the value of output
resistance RL and the drain resistance (RD)
is equal; then the value of current gain.
I0
Ai = is equal to:-
Ii
The small signal voltage gain of the
A. 4 B. 2
MOSFET amplifier is |Av|_______.
C. 0.5 D. 0.25
(rounded up to two decimal value)

75
www.gradeup.co

24. In the circuit shown, transistors are 27. For the circuit shown. Choose the correct
characterized by |Vt| = 2V, K’W/L = option(s) if IDSS = 6 mA and Vp = –4 Volt?
1mA/V , and λ = 0. If both of the
2

transistor is in saturation region so the


value of V2 is ………………

A. ID = 2.34 mA
B. ID = 3.75 mA
A. 6V B. 2V C. VDS = 7.98 V
C. 4V D. 5V D. VDS = 3.75 V
25. In the circuit shown, below, the transistor 28. For the given MOSFET
parameter are as follows?
ID = 0.5 mA,
Threshold Voltage VTN = 1V
VD = 0.6 V
Conduction parameter Kn /2= 0.4 mA/V
The NMOS transistor have
VY = 0.7 V, μncox = 120 μA/V2
w = 33 μm
L = 1.1 μm
(Neglect the channel length modulation)

So, the value of VDS is.


A. 6.40V B. 5.89 Volt
C. 3.20V D. 2.40 Volt
26. Consider a n-channel MOSFET with W =
15μm L = 2μm, and Cox = 69 nF/cm2,
Assume that in the non-saturation region
with VDS = 0.1V, the drain current is 35 μA
for a gate-to-source voltage of 1.5V, and
75μA for a gate to source voltage of 2.5V Choose the correct option(s) regarding the
compute the threshold voltage of the NMOSFET parameters?
MOSFET form the given data. A. VGS = 1.22 V
(Use small-VDS approximation in the drain B. RS = 3.65 kΩ
current equation). C. RD = 4.8 kΩ
A.0.3V B. 0.1V
D. Circuit works in triode region.
C. 0.935V D. 0.625V

76
www.gradeup.co

ANSWER

1. -8.01 2. C 3. D 4. A 5. 4.5 6. 1.36 7. 69.44 8. 2.175 9. B 10. D

11. B 12.-8.01 13. C 14.B 15. B 16. D 17. D 18. B 19. 0.424 20. 0.5

21. D 22. 1.43 23. 6.12 24. B 25. B 26. D 27. A,C 28. A,C

SOLUTION

1. Ans. – 8.01 VO
Ro = = rO rs
gm = 1mA/V, vi
ro = 50k = 100k 500
Draw the equivalent small signal circuit for
R o = 0.498K
the given amplifier
3. Ans. D.

Apply KCL at the source

Vo V V − Vin V
+ o + o
1.2k 1k 1M 30k
( )
− 0.002 Vgs + o = 0

300k || 60k
vgs = v AC equivalent circuit
300k || 60k + 2k in
= 0.9615 vin
vo = – gmvgs × 10k || 50k
= – 1mA × 0.9616 × [10k || 50k]
= – 8.01
2. Ans. C.
gm = 2ms
ro = 100k
1 1
rs = = = 500
gm 2 ms
Vo V V − Vin V
Draw small signal equivalent circuit for + o + o − 0.002  Vin − Vo  + o = 0
1.2k 1k 1M 30 k
finding ro  1 1 1 1 
 Vo  + + + 0.002 +
1.2k 1k 1M 30k 
1 
= Vin  + 0.002
1M 

⇒ Vo[0.003867] = Vin[0.002001]

Vo 0.002001
Voltage gain = = = 0.517
Vin 0.003867

77
www.gradeup.co

4. Ans. A. And Remaining circuit.


For figure (a) → VGS = VG – VS = 0 – V1 M3 and M4 in saturation region
= –V1

   ( V − VT )
2

ID = Kn   GS
L  2
2
 −V − 1
 10A = 0.4  10−3  1
2
⇒ V1 = –1.22V
For fig (b)
VGS = VG – VS = 0 – V2 = –V2 ID3 = ID4
K(8–V0–Vt)2 = K(V1–Vt)2
   ( VGS − Vt )
2

ID = kn   (8–V0–Vt) = (3.5 – Vt)


L  2
V0 = 8–3.5
2
 −V2 − 1
 100 A = 0.4  10−3 V0 = 4.5 Volt.
2
6.
V2 = –1.7V
By Voltage division Rule.
So, V1 > V2
18  22M
And max (V1, V2) = – 1.22V VG =
22M + 22M
5.
VG = 9V
In given circuit, all the MOSFET are
And
identical.
VGS = VG – ID RS
VDD
VGS = VG –
3
VGS = 9 – 6 = 3V
And we know that if VDS ≥ VGS – Vt MOSFET
is in saturation region.
I2 = 1.5mA. 1  W
( V − Vt )
2
ID = k
V1 = 8 – 3KΩ × I2 2 n  L  GS

V1 = 8 – 3 × 103 × 1.5 × 10–3 1  220  2


ID =  60    (3 − 1) A
V1=3.5v 2  6 

ID = 4.4 mA
And. VD = VS = ID RS = ID RD
6 = 4.4 mA. RS
6
RS =  103
4.4
R S = RD = 1.36k

78
www.gradeup.co

7. 8.

The drain current of the two transistors can For dc analysis, we redraw the circuit as,

be expressed as

I0 (W / L)2
=
IREF (W / L)1
(W / L)2 = 5(W / L)1

And from the given circuit.

In the circuit, the transistor is an n-channel


MOSFET. For the MOSFET we have
IG = 0
So, VD = VG
VDS > VGS – VTh.
i.e. the MOSFET is operating in saturation.
V1 = VDD – IREF × RD So the train current, and transconductance

V1 = 3 – 2mA × 1kΩ are defined as.


gm = 2Kn. (VGS – Vt)2
V1 = 1V
ID V − Vt
And V1 = VG. = GS
gm 2
VDS > VGS – Vt, transistor is in saturation 0.8 mA VGS − 0.6
=
10 ms 2
region.
0.16 = VGS − 0.6VGS = 0.76
Saturation Current.
Ans VG = VD.
W
ID = K1n   ( VGS − Vt )
2
At the drain terminal we have.
 L 1
2.5 − VD
A
W ID =
ID = 400 2    ( VGS − Vt )
2
RD
V  L 1
2.5 − 0.76
W RD =
2mA = 400  (0.6)2    10−6 0.8mA
 L 1
RD = 2.175k
W
 L  = 13.88 9. B.
 1
By drawing the small signal equivalent
W W
And   = 5   = 69.44 circuit by de activating all DC supplies, we
L
 2  L 1
get

79
www.gradeup.co

V0 = –gmVgsRD
From figure, V0 = –(gmRD)Vin = –[1 × 10–3 × 10 × 103]
−Vgs × 3 sin ωt × 10–3
Rin = and Ii = –gmVgs
Ii V0 = –30 sinωt mV
−Vgs 1 11. B.
 Rin = =
−gmVgs gm

10. D.
First applying the D.C. analysis, we have

No DC gate current
IG = 0
VGS = VDS (saturation region)

 W  ( VGS − Vt )
2

ID = kn L 
Now, assuming MOS to be in saturation   2
2
region 15 − 10ID − 1.5
ID = 0.25 
nCox W 2
( VGS − VT ) = 0.5  10−3(3 − 2)2
2
ID =
2L ID = 1.06 mA
ID = 0.5 × 10 –3
A VD = 15 – 10.6 = 4.4V
VDS = 10 – 10 × 10+3 × 0.5 × 10–3 VGS = VDS = VD = 4.4V
= 10 – 5 = 5V The maximum allowable input signal is
VDS > VGS – VT determined by the condition to keep the
5>3–2 MOSFET in saturation
Hence, our assumption was true. Vgs – Vt = Vds
nCox W VGS + Vgs − Vt = VDS + Vds
Now, gm = 2 .ID
2L (DC) (AC)
−6
= 2 0.5  0.5  10
⇒ 4.4 + Vin – 1.5 = 4.4 + Vo
gm = 1 mA/V ⇒ Vin – 1.5 = Vo
Now, drawing the small signal equivalent ⇒ Vin – 1.5 = –3.3 Vin
circuit, we get ⇒ 4.3 Vin = 1.5

80
www.gradeup.co

1.5 −2.56 + 5
Vin = = 0.34V ID2 = = 2.44mA
4.3 1k

( )
2

AV
V
= O = −gm ro || RD||RL  = −3.3    VGS1 − Vt
ID1 = ID2 = kn  
Vin L  2
2
 VA  −3 5 − V1 − 1 V2 − (−5)
ro = = 47k   2  10 =
 ID  2 1k
2
 ID  4 − V1 
−3
gm = = 0.725mA / V   2  10 = 2.44  10−3
 VGS  2

12. ⇒ V1 = 2.44 or 5.56 V

gm = 1mA/V, ro = 50k From the circuit V2 < V1 < 10 and V1 < 5V

Draw the equivalent small signal circuit for = 2.44 V (acceptable)

the given amplifier 14. B.


For Vin = +2.5V

300k || 60k
vgs = v
300k || 60k + 2k in
= 0.9615 vin
vo = – gmvgs × 10k || 50k
= – 1mA × 0.9616 × [10k || 50k] NMOS

= – 8.01 VGS= VG – VS = 2.5 – (–2.5) = 5V

13. C. VGS – Vt > VDs (triode region)

VGS2 = VG − VS = 0 − V2 = − V2 [∴ VDS (max) is supply voltage = 2.5 V]


PMOS
   ( VGS − Vt )
2

ID = kn L  VSG = VS – VG = 2.5 – 2.5 = 0 V


  2
Transistor is in cut off
( −V2 − 1)
2
V − (−5)
 2 = 2  10−3
1k 2    V2 
2
IDN = ID = kn   ( VGS − Vt ) VDS − DS 
 V2 + 5 = V2 + 2V2 + 1  L   2 
 V22 + V2 − 4 = 0
Assuming VDS is small
By solving  
IDN = ID = kn   ( VGS − Vt ) VDS 
V2 = 1.55V and –2.56V L 
As VG = 0V IDN = 1  10−3 (5 − 1) ( VD − VS ) 
Therefore V2 = 1.55V = 10−3 4 ( −ID ) 10k − (−2.5) 
Vgs<0 → wrong
= 1[4[–10ID + 2.5]]
V2 = –2.56 V
We get ID = 0.244 mA
Vgs>0 → acceptable
VD = –ID (10k) = –2.44V

81
www.gradeup.co

15. B. 17. D.
Small signal analysis of given figure as: For dc analysis, we redraw the circuit as

This is n-channel MOSFET. The drain

current for the MOSFET is defined as

1 W
( V − VTh ) (let assume
2
Vo = –gmVGSRD Vi = VGS ID =  C
2 n ox L GS
Vo MOSFET is in Saturation region)
AV = = gm RD [given AV = 5]
Vi
1
 200 (278) ( VGS − 0.4 )
2
Or 0.5m =
5 2
 gm = = 3.57 mA/V [given RD = 1.4 kΩ]
RD
Or VGS = 0.534V
In saturation region
Since, source is connected to ground, i.e.
ID = Kn(VGS – VT)2
VS = 0
ID
= 2Kn ( VGS − VT ) From above data
VGS
Vds>Vgs-Vth ( our assumption is correct)
ID
= 2Kn So, the voltage across R2 is
Kn
VG = 0.534 V = VGS
= 2 KnID
Since the gate current in MOSFET is IG = 0,
 1  so current ID/10 flows through
 Kn = 2 ncox L 
 
Resistances R1 and R2. Therefore, we have
 
gm = 2nCox   ID VG − 0 0.534
L  R2 = = = 10.68 k
1 0.05m
lD
10

 3.57  10−3 = 2  200  10−6   10−3
L Applying KVL in loop 1, we get
  11 1
 L = 31.9 1.8 − I (2k) − I (R + R2 ) = 0
  10 D 10 D 1
16. D. Or R1 + R2 = 14k
All statements are correct. Thus, R1 = 3.32 kΩ

82
www.gradeup.co

18. Ans. B. 20.


For Ac Analysis capacitor are shorted, the In the given circuit.
dc sources are replaced by their internal VTN = 0.5V

resistance. Kn = 2mA/V2.
Id= 1 mA
And,
gm = 2KnId

gm = 2  2  1(mA / V2 ) = 2mA / V
And equivalent circuit of MOSFET.
|Av| = gmRD
AV 8
gm = = = 10 mA/V
RD 800

Again, we define the trans conductance as.

gm = 2KnID
W
gm = 2nCOX   ID
L 
W 1 1
10  10−3 = 2  120  10−6    0.8  10−3 Zi = = = 0.5k
L  gm 2mA
W 21. D.
 L  = 520.83
  Drawing the small signal equivalent of the
19. above circuit, we get
We draw, the small signal model of given
circuit as

 RD 
Now, I0 = −   gmVgs 
 RD + RL 
Applying KCL at the input, we have
Corner frequency due to coupling capacitor
Vgs
CC is defined as] Ii + gm Vgs + =0
R si
1  R si 
fL =  Vgs = −Ii  
2R eqCC 1 + gmR si 
Req = (7.2 + 7.8)kΩ = 15 kΩ I  RD   R si 
 Ai = 0 =    gm  
Ii R
 D + R L  1 + g R
m si 
So the coupling capacitor is
1
Now, RD = RL andR si =
1 gm
=
2  15  25  103 I0 1 1 1
Thus, Ai = =  = = 0.25
CC = 0.424 μF. Ii 2 2 4

83
www.gradeup.co

22.
The amplifier can be modelled as shown in
fig below
RL
Now, Vout = A v0 Vin 
RL + R 0

Where, AV0 = Voltage gain when RL = not


connected
From the circuit, we have output voltage,
V0 = -gmVGS × 10 k
At the input VGS = Vi
V0 = -gmVGS × 5 × 103
V0
AV = = −1.22  5  103
Vi
90
Thus, A VO = = 10V / V = – 6.12
9
|Av| = 6.12
Vout RL
= A 0 
Vin RL + R 0 24. B.
Considering the upper MOSFET
70 10  103
 = 10 
8 10  103 + R 0 knW
(10 − V1 − 2)
2
I0 =
2L
⇒ R0 = 1.43 kΩ
1
(10 − V1 − 2 )
2
23. 2=
2
For the MOSFET transconductance is given
by.
2= (
1 2
V − 16V1 + 64
2 1
)
Id
gm = V12 − 16V1 + 60 = 0 V1 = 6V or10V
VGS
We choose 6V for which VSG > |Vt| to avoid
= 2 kn (VGS - VTN)
cut off.
gm = 2 × 0.75(VGS - VT)
For the lower MOSFET.
VT = 1V
And, I = kn(VGS -Vt)2 Kn W
(6 − V2 − 2)
2
ID =
2L
0.5 × 10-3 = 0.75 mA/V2 (VGS - Vt)2
1
( 4 − V2 )
2
VGS – Vt = 0.816 2=
2
gm = 1.22 mA/V
1 V22 − 8V2 + 16 = 4
And r0 = =
D V22 − 8V2 + 12 = 0
In AC analysis, the dc sources are
V2 = 2V V2 = 6V
replaced, by their internal resistance and
We choose 2V for which VSG > |Vt| (to
capacitor, are shorted. So we have the
avoid cut off)
small signal equivalent circuit as.
option B. is correct.

84
www.gradeup.co

25. B. Drain, current for the MOSFET circuit is,


For the analysis, we have the circuit 4.66 − 2.37
ID = = 0.76mA
3K
And,

By using voltage Divider Rule


14K
VG =  12 = 4.66V
22K + 14K
Applying KVL in train source loop
10 – ID × 4K – VDS – ID × 2K = 0
MOSFET Gate current IG = 0. VGS = VG –
VDS = 12 – ID (5k + 3k)
V S. VDS = 5.89
VG − VGS VDS>VGS-Vth (Assumption is correct)
ID =
Rs
26. D.
Assume that MOSFET in saturation region. Given

ID = kn ( VGS − Vt ) / 2 W = 15μm L = 2μm Cox = 69nF/c.m2


2

VDS = 0.1V ID1 = 35μA, VGS = 1.5V


VG − VGS
= kn ( VGS − 1) / 2
2
ID2 = 75μA VGS2 = 2.5μA
RS
Given MOSFET is in non-saturation, region.
4.66 − VGS
= 0.4 mA / V2 ( VGS − 1)
2
So
3k
 V2 
ID = K  VDS ( VGS − Vt ) − DS 
4.66 − VGS = 1.2
mA
V 2 (
 103 VGS
2
)
− 2VGS + 1  2 

2
VDS
V 2 And VDS is small 0
3.88 − GS = VGS − 2VGS + 1 2
1.2
2
Drain current equation for VGS = 1.5V
VGS − 1  16VGS − 2.88 = 0
35 × 10–6 = K[0.1(1.5 – Vt)] … (i)
Solving the quadric equation we obtain. And drain current equation for Vgs = 2.5V

1.16  (1.16)2 + 4  2.88 75 × 10–6 = K[0.1 (2.5 – Vt)] … (ii)


VGS =
2 ID1 35 0.1 (1.5 − Vt ) 
= =
VGS = 2.37V, −1.21V ID2 75 0.1 (2.5 − Vt ) 

For n-channel MOSFET VGS is always 0.25 × 7 – 7Vt = 0.15 × 15 – 1.5 Vt

positive. So, we have 0.8Vt = 0.5

VGS = 2.37 V Vt = 0.625

85
www.gradeup.co

27. A & C 2  0.5  1.1  1000


(VGS − VT )2 =
2 120  33
1 − VGS 
ID = IDSS   (VGS = –VT) = 0.52
 VP 
VGS = 0.52 + 0.7
2 2
 1.5  5
ID = 6 mA 1 −  = 6  VGS = 0.52 + 0.7
 4  8
ID = 2.4 mA −1.22 + 3
RS = k
0.5
VDS = VPD – IDRD = 15 – 3 × 2.34 = 7.98 V
28. A & C
1.78
RS = = 3.56 k
0.5
Since VD > VS, Hence circuit works in
3 − VD 2.4
saturation region RD = = = 4.8 k
ID 0.5
1 w
so, 30 = ncox (VGS − VS )2
2 L



86
www.gradeup.co

87
www.gradeup.co

88
www.gradeup.co

89
www.gradeup.co
Chapter

8 Multi-Stage Amplifiers

1. A single stage amplifier has a mid-band

gain Am = 500, lower and upper cutoff

frequencies fL = 50 hz and fH = 1 KHz of a

negative feedback with feedback gain

1
= is used then lower and upper cut
100

off frequencies will be respectively :

A. 300 Hz, 0.166 Hz 4. The overall voltage gain of four identical


B. 300 Hz, 6 Hz stage of an amplifier. If each having a gain

C. 8.33 Hz, 0.166 Hz of 20 is ______ dB.


5. If the lower 3dB frequency of an amplifier
D. 8.33 Hz, 6 KHz
is 100 Hz then the lower 3dB frequency of
2. Consider the given amplifier circuit shown
the cascade structure of three such
below. The circuit has following parameters amplifiers will be,
rπ = 0245 kΩ , Vt = 26mv A. 50.98 Hz B. 196.14 Hz
what is the value of 3 – dB frequency C. 155.37 Hz D. 64.35 Hz

f1(Hz) of the amplifiers due to coupling 6. An ideal voltage amplifier has voltage gain
of –1000 and has 1 pF capacitor connected
capacitor CC ?
between input and output terminal. If the
voltage source feeding amplifier has
internal resistance of 100Ω, the upper 3-
dB frequency will be ______ MHz.
7. Find the transfer function of the amplifier
shown assume that rds = ∞ & gm =
1ms for all MOSFET and R = 10KΩ and
C = 100 nF

3. Consider the given circuit having the lower

cutoff frequency due to coupling capacitor

CC is fi = 20Hz, then find the value of

coupling capacitor CC in ______ μF

90
www.gradeup.co

1000 −10 10. For the JFET-BJT Darlington pair. Find


A. B.
 S 
3  S  voltage gain AV. If β = 99 and rπ = 1 kΩ for
 3 + 1  3 + 1
 10   10 
BJT and µ = 60, rds = 30 k for JFET.
−1000 −100
C. 3
D. 2
 S   S 
 3 + 1  3 + 1
 10   10 
8. For the multistage amplifier given below,
o
find its voltage gain ?
vin

(considering large β)

11. For the Darlington amplifier shown in


figure below as:

−R C
A.
1 / gm2 + RE || r1

−R C Both Q1 and Q2 are identical with h


B.
1 / gm1 + RE || r2
parameter hfc = 100, RE = hie = 1 kΩ. If
−R C z parameter of the circuit diagram is given
C.
1 / gm1 + RE || r1
as
−RC  V1  Z11 Z12  i1 
D.
1 / gm2 + RE || r2  =  
 V2  Z21 Z22  i2 
9. The voltage VD (in V) for the given network
Z11 − Z21
shown below is. Then is
Z12

A. 100 B. 50
C. 75 D. 25
12. The two-stage system of figure employs a
transistor emitter-follower configuration
prior to a common base configuration to
ensure that the maximum percentage of
the applied signal appears at the input
terminal of the common base amplifier. In
figure the no load values are provided for

91
www.gradeup.co

each system, with the exception of Z I and 15. Read the following option and choose the
Z0 for the emitter follower, which are the correction option(s) if lower cut-off
loaded values. frequency of each amplifier is 50 Hz.
A. For a Cascade of 2 non interacting
amplifier, the overall cut off frequency
is 77.7 Hz.
B. For a cascade of 2 interacting
amplifier, the overall cut-off frequency
The total gain of the system is
is 77.7 Hz
A. 96.54 B. 0.714
C. For a cascade of 2 non-interacting
C. 104.21 D. 89.11
amplifier, the overall cut-off frequency
13. A frequency compensated OP-amp has an
is 77.5 Hz
open loop transfer function with signal pole
D. For a cascade of 2 interacting
106
A= . It works as non-inverting amplifier, the overall cut-off frequency
 S 
 1 +
 10  is 77.5 Hz.
16. For the below high frequency π model,
amplifier. The unity gain frequency is
Read all the options and choose the correct
Wt and 3 dB band width (W3dB) of the
option(s)?
closed loop non- inverting amplifier is ____
KHz.

If the value of Thermal voltage is 26 mV


and collection current is 1 mA. Base
current is 10 μA.
A. β cut off frequency of the BJT will be
14. Identical non interacting amplifier having
10.2 Hz
cutoff frequencies of multiple stage circuit
B. Unit gain frequency of the BJT will be
will be>
10.2 Hz
(More than one option may correct)
C. β cut-off frequency of the BJT will be
fL = lower cut-off frequency
1.02 kHz
fH = Higher cutoff frequency
D. Unity gain frequency of the BJT will be
A. fL = 31 Hz
1.02 kHz
B. fH = 31 Hz
17. For the below amplifier having hfe = 120 hi
C. fL = 12.9 Hz
= 2.2 kΩ.
D. fH = 12.9 kHz
Choose the correct option(s)?

92
www.gradeup.co

A. Medium frequency voltage gain will be


–163.64
B. Overall input Resistance will be (Seen
by Vi) 2.18 kΩ
C. Lower 3 dB frequency will be 6.63 Hz
D. Ro (Output Resistance) will be 3 kΩ

ANSWER

1. D 2. 13 3. 0.476 4. 104.08 5. B 6. 1.58 7. C 8. B 9. A 10.C

11.A 12. D 13. 100 14. A, D 15.B,C 16. A,D 17. A,B

SOLUTION

1. D. 2.
Am = open loop gain = 500 So, we have to find the 3dB frequency f1,
fL = 50 Hz, fH = 1 KHz due to CC. so, we will short remaining
If feedback is used, then gain decreases. capacitances
So, small signal equivalent circuit is

τ1 = rc Cc
The lower cut off will decrease and upper rc = (1.9k || 9.1 k || 0.248k) + 400Ω
cut off frequency will increase by using = 211.96 + 400
negative feedback. = 611.96 Ω
fL τ1 = rc CC
fL = [∵ 1 + βAm = 6]
(1 + Am ) = 611.96 × 20 × 10–6

50 = 0.0122
= = 8.33Hz
6 1
f1 =
21
fh = fh (1 + Am )

= 103 × 6 = 6 KHz f1 = 13.00 hz

93
www.gradeup.co

3. 6.

Here device parameter is not given and we The amplifier can be like

also do not require as the coupling

capacitor is on collection side so, small

signal equivalent circuit for this capacitor

Cm and Cn are miller capacitors


Cm = C[1 – AV] = 1 pF [1 + 1000]
= 1001 pF
 1   1 
Cn = C 1 −  = 1pF 1 −   1pF
 A V  1000 
so, τ = ro co

τ = (10k + 6.7k) 1 1
So, fH = =
2R sCm 2  100  1001  10−12
1
f = = 1.58 × 106 Hz
20
7. C.
1
20 = Let us consider the first stage of the
2  16.7  103  C0
amplifier & then draw the AC equivalent
1 ckt.
C0 = 3
2  16.7  10  20 As given capacitors are small capacitors so
= 0.476 μF their impact will come into picture

4.

Given = Av = 20

Overall voltage gain of each

Stage = (Av)n
Drawing email signal AC equivalent ckt.
= (20)4

Gain in dB = 20 log (20)4 = 104.08dB

5. B.

FL
fL = 1
1/n
2 −1 z=R
cs
100 1
= R
1
= cs
23 −1 1
R+
FL’ = 196.14 Hz cs

94
www.gradeup.co

R 9. A.
z=
1 + RCS 2
 VGS 
R ID = IDSS 1 − 
 Vo(s) = −gmVgs  VP 
1 + RCS
2
V (s) −gmR  V 
 o =  ID = 8 1 − GS 
vin(s) 1 + RCS  4 

When three such stage are cascaded Because VGS = – IDRS

Vo (s)  −gmR 
3 = –2.4ID
= 
Vin(s)  1 + RCS  −VGS
ID =
2.4
−1000
 TF = 3 Upon solving we get,
 S 
 3 + 1 VGS = –2.544 V1 –6.29V
 10 
VGS is not greater than the Vp(considering
8. B.
Draw the small signal equivalent of this magnitude)

multistage amplifier, so VGS = –2.544V and ID = 1.06mA


For transistor

IE IC ID = 1.06 mA
0.06
IB = = 0.01325mA
80

VD = 16 – 470 × 0.01325 — 0.7


VD = 9.07V
10. C.

re1
vbe1 = vin
re1 + RE r2

V0 = – gm1VbeRc
re1  vin
= −gm1   RC
re1 + RE || r2


sin ce, re =
gm

1
v0 gm1
So, = −gm1   Rc
vin 1
+ RE r2 From figure:
gm1

Since large β so, α = 1 Vo = ib + 99ib  2k

v0 −R C = 100 ib [2k]
=
vin 1 Vin = Vgs + ib [1k] + 100ib [2k]
+ RE || r2
gm1

95
www.gradeup.co

KVL in loop We get


ib [30k] − 60 Vgs + ib [1k] + 101ib (2k) = 0 Z11 = (1 + 1 × 101 + (101)2) k
ib [30k + 1k + 101(2k)] = 10.3 MΩ
Vgs =
60 Z21 = (101)2 × 1 kΩ = 10.2 MΩ
Vgs = ib [3.883 k] Z12 = Z22 = RE = 1 kΩ
Vin = ib [3.883k] + ib [1k] + 100ib (2k)
Z11 − Z21 (10.3 − 10.2)  103 k
Vin = ib [204.88k] =
Z12 1k
Vo 100ib [2k]
Voltage gain = A V = = = 0.976 = 0.1 × 103 = 100
Vin ib [204.88k]
12. D.
11. A.
For the emitter-follower configuration the
Drawing the hybrid model of two transistor
loaded gain is
together.
Zi2 25
V01 = A V = (1)Vi1
Zi2 + Z01 VNL i1 25 + 10
V01 = 0.714Vi1

V01
And, A vi = = 0.714
Vi1

For the common base configuration.


RL
V02 = A
RL + R OL VNL
9.6k
Given hie = 1 kΩ, RE = 1kΩ Vi2 =  200Vi2
9.6k + 4.6k
Ib2 = I1 (1 + hie ) → from circuit V02 = 135.21Vli2

Current through RE V02


A V2 = = 135.21
Vi2
IRE = Ib2 + hfc Ib2 + I2
A VT = A V1  A V2
→ apply KVL to the loop containing
= 0.714 × 135.21
V1, RE etc we get
= 96.54
(
V1 = hie Ib1 + hie Ib2 + Ib2 + hfc Ib2 + I2 RE ) Zi1
A VS = A VT

( ) ( ) 
2
V1 = hie + hie 1 + hfc + 1 + hfc RE  I1 + I2RE Zi1 + R S
 
→ apply KVL to the loop containing V2 12k
=  96.54
12k + 1k
(
V2 = Ib2 + hfC Ib2 + I2 RE ) AVS = 89.11
( )
2
= 1 + hfC I1RE + REI2 Option (D) is correct.
13.

( ) ( ) 
2
 V1  hie + hie 1 + hfc + 1 + hfc RE RE  I 
1 AO 106
 =  I  A= =
 V2  
( )
2
1 + hfc RE RE   2  1+
S
1+
S
  W3dB 10

96
www.gradeup.co

Ao = open loop gain = 106 IC 1 mA 1


Im = = =
Gain bandwidth product = 106 × 10 = VT 26 mA 26
107 rad/sec Im 1
fT = =
Gain of closed loop 2(c + c ) 26  2  6  10−6
 R   990 
A V = 1 + f  = 1 + = 100 106
 R   10  fT = = 1.02 kHz
312
∴ 3dB bandwidth of closed loop system
fT f 1.02 kHz
107 f = = T =
W3dB closed loop =  I 100
gain of closed loop C
IB
= 105
14. A & D f = 10.2 Hz

fL1 20 17. A & B


fL = =  31 Hz
21/2−1 0.645

fH = fH1 21/2−1 = 20 kHz  0.645 = 12.9 kHz

15. B & C
For interacting amplifiers

fL = 1.1 tl21 + tl22 + ... ℎ𝑓𝑒 (6||6) 𝑘𝛺 120 × 3


𝐺𝑎𝑖𝑛 = − =−
ℎ𝑖𝑐 2.2
fL = 1.1 502 + 502
Gain = –163.64
For non-interacting amplifier Ri = RB || hie = (250 || 2.2) kΩ
fL1 Ri = 2.18 kΩ
fL = N = 2 Here
1/N−1
2 1 1
fL = =
50 50 2(R L + R L )C 2  (6 + 6)  0.5  10−3
fL = = = 77.5 H2
21/N−1 0.645
000
16. A & D fL = = 26.52 Hz
12
β cut off frequency fs = fI
Ro = 6 kΩ
fT = unity gain frequency



97
www.gradeup.co

98
www.gradeup.co
Chapter

9 Feedback Amplifiers

1. The feedback used in the circuit shown 4. An amplifier have a gain of 80 without
below can be classified as feedback and its input and output
resistance is given by 2kΩ and 5 kΩ. The
amplifier using series-shunt negative
feedback with a feedback factor of 0.05.
The input and output resistance of
feedback system is
A. Rin = 1 k Ω R of = 5 k Ω
B. Rin = 10 k Ω R of = 5 k Ω
A. Shunt – series feedback
C. Rin = 10 k Ω Ro = 1 k Ω
B. Series – shunt feedback
D. Rin = 15 k Ω Ro = 7 k Ω
C. Shunt – shunt feedback
5. Consider the following statements
D. Series – series feedback
regarding the trans-impedance of the
2. Consider the following feedback amplifiers
Amplifier:
whose open loop gain is very high, the
closed loop voltage gain is ______ (i) The input impedance of the circuit
decreases and output impedance of
the circuit increases.
(ii) Amplifier have a shunt-shunt topology.
(iii) At input side current mixed and
voltage sampled at output side.
(iv) Bandwidth of transimpedance amplifier
Increased.
3. If three amplifiers having the same Which of the above statement is /are
bandwidth are cascaded, the bandwidth of
incorrect?
the resulting amplifier will be
A. only (i)
A. Better than that of each stage
B. (i) and (ii)
B. Worse than that of each stage
C. (ii) and (iii)
C. Same as that of each stage
D. (i), (ii) and (iii)
D. None of the above

99
www.gradeup.co

6. Assertion (A): Positive feedback theory is A. series-shunt B. shunt-shunt


applied for unstable system. C. series-series D. shunt-series
Reason (R): Oscillator is a system that 10. An amplifier has an open loop gain 600.
output is infinite at Zero input.
And a feedback 0.04 . If open loop gain
A. Both A and R are true, and R is the
changes by 15% due to change in closed
correct explanation of A
loop gain is _______(Rounded up to two
B. Both A and R are true, but R is not the
decimal value)
correct explanation of A
C. A is true, but R is false 11. Consider an op-amp circuit given in Figure.

D. A is false but R is true


7. Consider the amplifier circuit shown find
feedback factor β (in %)?

If Op-Amp gain is 1000 and resistance


R1 = 1.8 kΩ and R2 = 200 Ω. So amplifier

8. Given the basic feedback amplifier (op- gain of the circuit is ______ V/V (Rounded
amp) has A = 104, Rin = 105Ω Ro = 1 kΩ. up to two decimal value).
Vo 12. The given circuit has a feedback factor of
Find
VS (Assume Beta value to be very high)

9. Identify the type of feedback in the


amplifier given below

A. –0.4 B. 0.4
C. 2.5 D. –2.5
MSQs:
13. Which of the following feedback (s0
inverse(s) the overall input Resistance of
an amplifier?
A. Voltage series B. Voltage shunt
C. Current series D. current shunt

100
www.gradeup.co

14. Choose the correct option(s) regarding


voltage Buffer?
A. Buffer is a circuit which provides
impedance matching between source &
Load
B. Buffer is a circuit, used to transfer
Then which of the following statement
maximum power form source to load.
is/are correct?
C. Input impedance is very large for
A. Input voltage applied to basic
voltage Buffer
transconductance amplifier is 5 mV.
D. Output impedance is very low for
B. Forward gain (without feedback) of
voltage Buffer
transconductance amplifier A is 2
15. An amplifier has high frequency gain given
(A/V).
AM
by AH = where AM = 1000 and ω2 = C. Feedback gain (β) is 9.5 (V/A)
j
1+ D. Overall gain 0.1 A/V
2
17. For shunt-shunt feedback circuit using an
104 Rad/sec.
ideal trans-resistance amplifier consider
Choose the correct option(s) if we increase
IS = 100 μA
the upper corner frequency ω 2 to 105
If = 95 μA
Rad/sec.
Vo = 10 V
A. Feedback factor required is 10
Then which of the following statement
B. Feedback factor required is 0.009
is/are correct?
C. New overall gain of amplifier is 200
A. Input current through amplifier is 5 A
D. New overall gain of amplifier is 100
B. Open loop gain is 2 × 106 V/A
16. A series feedback amplifier using a
C. Feedback factor is 9.5 A/V
transconductance amplifier operates with
D. Feedback factor is 9.5 μA/V.
VS = 100 mV, Vf = 95 mV, and Io = 10 mA,
respectively.

ANSWER

1. C 2. 151 3.B 4.C 5.C 6.A 7.A 8. -868.2 9. A 10. 0.6

11. 9.99 12.D 13.A,C 14.A,B,C,D 15. B,D 16. A,B,C,D 17.B,D

101
www.gradeup.co

SOLUTION

1. C. 4. C.
The small signal equivalent circuit of given For a series-shunt topology, the input
amplifier is shown below. Here the resistance is given by.
feedback circuit samples the output R inf = Ri (1 + Aβ)
voltage and produces a feedback current and output resistance is given by
Ifb which is in shunt with input signal. RO
R of =
(1 + A)
A = 80 β = 0.05
R inf = 2 (1 + 80 × 0.05) K = 10Ω
5K 5K
R of = = = 1K
(1 + A) (1 + 0.05  80)
5. C.
In the input side of transimpedance
So, this is a shunt-shunt feedback Amplifier current mixed in shunt and at
configuration. output side voltage sampled in shunt
2. therefore these is also known as shunt -
Consider only feedback network shunt Amplifier.
In transimpedance Amplifier input and
output resistance decreases.
Bandwidth of feedback Amplifier Always
increases.
6. A.

Vf = βVo (β = feedback factor)


Vf 100 1
= = =
V0 15000 + 100 151

Closed loop voltage gain


AV 1
A Vf = = ( A V  )
1 + A V  If Aβ = 1 So,

 A Vf = 151 A V
= 0
1 − AB Vi
3. B.
V0 A
A single stage amplifier has finite = =
Vi 0
bandwidth. If you cascade multiple stages,
Output is unbounded for bounded input
more gain is achieved, but additional
and oscillator is unstable system.
bandwidth limitations occur since each
So, option (A) is correct.
stage has finite bandwidth.

102
www.gradeup.co
7. A. 8.
Now, here on input side the feed network
i.e.,

KCL at node (1)


Vd − Vs Vd V − Vo
− − − d =0
Is coming in the input loop, so it is in 1k 100k 1M
series, so it is voltage mixing ⇒ Vo = –1011 Vd – 1000 Vs …(i)
Now, on output side, the feedback n/w KCL at node (2)
with 4R resistance is directly connected to Vo − 104 Vd Vo + Vd Vo
+ + =0
output Vo, 1k 1M 2k
So, it is voltage sampling, and it is in shunt | 50 |
 Vd = Vo …(ii)
So, feedback amplifies is 107
Voltage – voltage Substitute 2 in 1
Or series – shunt 1501 
Vo = −1011  Vo  − 1000VS
Now, the feedback network for finding  107 
feedback factor, for finding input loop Vo
 = −868.2
output voltage grounded and for finding VS
output loop, input current is zero open 9. A.
circuited so,

Rf is not linked directly to the input,

2R R therefore it is a series mixing


vf = vo  
6R R + 3R Rf is linked directly to the output, therefore

1 1 it is a voltage sampling.
v f = vo  
3 4 Hence it is voltage-series topology which is
also called series-shunt topology.
vf 1
=
vo 12

=100/12 = 8.33%

103
www.gradeup.co
10. 𝐴𝐻
𝐴𝐻 𝐹 =
1 + 𝛽𝐴𝐻
A
Given A = 600 β = 0.04 & = 15% A
A AH F =
1 + j/2
Change in closed loop gain A
1+
A f A / A 1 + j/2
=
Af 1 + A
A
AHf =
15 15 1 + A
= = % = 0.6% j
1 + 600  0.04 25 1+
2 (1 + A)
11.
A
A Overall gain =
Af = 1 + A
1 + A
Overall frequency = (1 + βA)ω2
Af = gain with feedback
104(1 + βA) = 105
β = Feedback factor
1 + βA = 10
R2 200
= = = 0.1 1000
R1 + R2 1.8  103 + 200 Overall gain = = 100
10
A = 1000
9
1000 1000 = = 0.09
Af = = = 9.99 1000
1 + 1000  0.1 101
16. A, B, C, D
12. D.
Input voltage to basic transconductance
V
Feedback factor β = f . amplifier,
Vo
Vin = VS – Vf
−ICRE = 100 mV – 95 mV
=
ICR c
= 5 mV
5k So, option A is correct.
=−
2k
From figure,
β = –2.5. A = Io /Vin
13. A & C 10 mA
= = 2 (A/V).
Feedback Rif Rof 5 mA
Voltage series Ri⋅ D↑ Ro/ D↓ So option B is correct.
Voltage shunt Ri/D↓ Ro/D↓ Vf 95 mV
= = = 9.5 (V/A)
Current series Ri⋅ D↓ Ro⋅ D↑ Io 10 mA
Current shunt Ri/D↓ Ro.D↑
So, Option C is correct.
14. A, B, C & D
A 2
15. B & D Af = = = 0.1 A/V
1 + A 1 + 2(9.5)
𝐴𝐻
For negative feedback factor 𝐴𝐻 𝐹 =
1+𝛽𝐴𝐻
So, option D is also correct.

104
www.gradeup.co
17. B, D A = 2 × 106 V/A

Input current through amplifier is So option B is correct.

Ii = Is – If = 100 μA – 95 μA = 5 μA Feedback factor

So option A is wrong. If 95  10−6


= =
Vo 10
Given output voltage is 10 V.
= 9.5 × 10–6 = 9.5 μA/A
V 10
So open loop gain A = o = So, option C is incorrect while option D is
If 5  10−6
correct.



105
www.gradeup.co
Chapter

10 Power Amplifiers

1. A class B push pull amplifier must deliver


10W of audio power to the output load. If
the output transformer is 80% efficient
then what is the minimum average
dissipation rating required for each
transistor?
A. 3.4 W B. 1.7W
C. 2.5W D. 5W A. 170 B. 196
2. A class B push-pull amplifier is supplied C. 189 D. 200
with VCC = 50V. The signal brings the 5. Consider the circuit shown in figure below:
collector voltage down to Vmin = 5V. The
total dissipation from both transistors is
30W. Find the total input power?
3. Assertion (A): For class A operation for CE
transistor amplifier, the operating point
cannot be fixed near VCE = 0 point.
Reason (R): During the positive half of the
input cycle, the transistor may be driven
from “active region” to “saturation - The maximum ac output power and the dc
region”, thus losing the class A wave shape input power of the amplifier are
in the output. respectively: (Given Vcc = 20V)
A. Both A and R are true and R is the A. 12.5W, 15.5W
correct explanation of A B. 3.25W, 7.96W
B. Both A and R are true, but R is not the
C. 6.25W, 3.25W
correct explanation of A
D. 6.25W, 7.96W
C. A is true but R is false
6.
D. A is false but R is true
4. Consider the class AB stage amplifier
shown in figure with VT (thermal voltage)
= 25mV. If both transistors have reverse
saturation current Is = 10–15 A, the value of
quiescent current Ic is ______ μA (Assume
beta to be very high)

106
www.gradeup.co

In the ideal class B amplifier with


complimentary symmetry shown in the
figure,
VCC = 15V, RL = 10Ω
Find the maximum signal output power,
corresponding transistor power dissipation
and conversion efficiency?
A. 14.31 W, 3.07 W, 78.5% What is the value of RB in kΩ that locate
B. 15W, 3.07W, 81.2% the Q point at the center of the load line?
C. 14.31W, 2.07W, 62.5% MSQs:
D. 15W, 2.07W, 78.5% 10. Choose the correct option(s)
7. A single ended class A transformer coupled A. In class B Power amplifier, there is no
amplifier is to deliver a power of 50mW to cross over distortion
the load in the secondary. Given efficiency B. In class AB Power amplifier, there is no
of transformer is 70%, the transistor cross over distortion
chosen should have a power dissipation C. Any amplifier can be analyzed using h-
capability PT given approximate to? parameters
A. 35 mW B. 70 mW D. Any power amplifier can be analyzed
C. 180 mW D. 140 mW using h-parameters
8. A sinusoidal signal of 100Hz is applied to 11. For a class B amplifier, providing 20 V peak
an amplifier. The output current is I0 = 10 signal to 16 Ω load and power supply of 30
sin(628t) + 2 sin (1256t) + 1 sin(2512t) V, choose the correct option(s)?
what is the approximate percentage A. Output power will be = 12.5 W
increase in power due to distortion? B. Output power will be = 25 W
A. 4% B. 5.0% C. Efficiency of amplifier will be = 52.5%
C. 3% D. 6.5% D. Input power will be = 23.9 W
9. Consider a class A power amplifier circuit
given below:

ANSWER

1. B 2. 102.35 3. A 4. B 5. D 6. A 7. B 8. B 9. 11.3 10. B,C

11. A,C,D

107
www.gradeup.co

SOLUTION

1. B. 3. A.
Power delivered to load PL = 10 W For class A operation, operating point is in

PL mid of load line, so A is correct.


Pout (ac) =
output transformer efficiency R is also correct because if operating point
will be at VCE = 0, for positive half cycle,
10
= = 12.5W
0.8 transistor will be in saturation so wave
shape will be lost in the output. This is
P (ac) 12.5
Pin(dc) = out = = 15.625W correct reason.
 0.8
4. B.
Minimum average power dissipation
For dc analysis, we redraw the circuit as:
Pd = Pin(dc) − Pout (ac)
= 15.625 − 12.5 = 3.125W

Minimum average dissipation rating


required for each transistor

Pd 3.12
= = = 1.56 W
2 2
2.
VCC = 50V
Applying KVL in loop 1, we get
Vmin = 5V
VBE1 + VEB2 =1.3 V
Total power dissipation Pd = 30W IC I
Or VT ln + VT ln C = 1.3
∴Pd = Pin (dc) – Pout(ac) Is Is

2 (IC )Max (IS and VT are same for both transistor)


30 = VccIm −  Vcc − Vc min 
 2 IC
Or 2VT ln = 1.3
Is
2 I
30 =  50  Im − m [50 − 5]  1.3 
 2  
−15  22510−3 
So, Ic = 10 e = 196 μA
100 45 
30 =  − I
  2  m 5. D.
We redraw the given circuit as:
30
= Im
9.33

30
Im = = 3.215A
9.33

Total power input,


2 2
Pin (dc) = Vcc Im =  50  3.215
 

Pin (dc) = 102.35 W

108
www.gradeup.co

The maximum peak output voltage is:


VCC
Vout (peak) = VCBQ =
2
20
= = 10V
2
The maximum peak output current is
VCEQ
Iout (peak) =
RL
V0,max = VCC = 15V
10
= = 1.25A
8 V0,max 15
Im = = = 1.5A
So, we obtain the ac output power as RL 10
1 Im
Pout = I V I0,RMS = = 1.06A
4 CQ cc 2
1
=  1.25  20
4 Im
I1,DC = I2,DC = = 0.477A

= 6.25W
The dc input power for the amplifier is (a) P0,AC = I20,RMS  RL

ICQ Vcc = (1.06)2  10


Pdc =

= 11.24W
1.25  20
=
 PI,DC = 2 × VCC × IDC

= 7.96W = 2 × 15 × 0.477

6. A. = 14.31W

During positive half cycle of input Q1 is ON (b) Total transistor power dissipation
=PI,DC – P0,AC
and during negative half cycle of input,
= 14.31 - 11.24
Q2 is ON, so that we get a full-wave across
= 3.07W
the load RL.
P0,AC
(c) Efficiency =  =  100%
PI,DC

11.24
=  100%
14.31
η = 78.5%
7. B.
Pac = 50 mW at the secondary
KCL at the output node:
Transformer efficiency = η = 70%
I1 = I0 + I2
Input power to transformer
Following are the three current-waveforms
Pac 50  10−3
Pin = =
over one cycle  0.70

Pin = 71.42 mW

109
www.gradeup.co

Maximum efficiency of class A is 50% 10. B, C


Pac 71.42 In class B, there is cross over distortion,
Pdc = = = 142.85mW
 0.50 because both transistor of push Pull
Maximum transistor dissipation power amplifier are maintained in cut off state.
= 142.85 - 71.42 = 71.425 ≈ 70 mW
Hence, they take time to come to ON
8. B.
state.
For 1Ω resistance
So option A is incorrect.
Total power:-
In class AB, the transistors are kept in ON
(10)2 (2)2 (1)2
= + + W state at low current value or just ON State
2 2 2
= 52.5 W therefore cross over distortion is
Power due to distortion: eliminated.
2 2 So option B is correct.
(2) (1)
+ = 2.5 W
2 2 Any amplifier can be analyzed using h-
Hence, fundamental power = 50 W parameter model. But the analysis of
Percentage increase due to distortion:
Power amplifier is not about the voltage
(2.5/50) × 100= 5.0%
gain, input resistance and output
9.
resistance. The analysis of Power amplifier
For the Q-point to be at the center of the
load-line, we have is about power delivered by source and

VCC 12 received by load and efficiency. So, these


VCE = = = 6V
2 2 Parameters are not analyzed by using h-
12 − 6 parameter model.
And IC = = 60 mA
100
So, option C is correct, but option D is
60 mA
also, IB = = 1mA incorrect.
60
11. A, C & D
for dc-analysis, we redraw the circuit as
VP 20
I= = = 1.25 A
RL 16

2Im 2
INC = =  1.25 = 0.796 A
 
(Pi)dc = Vdc × Idc = 30 × 0.796 = 23.9 W

VP2 (20)2
Pout = = = 12.5 W
2RL 2  16
Applying KVL in loop 1, we have
12 - IBRB- 0.7 = 0 Pout 12.5
=  100 =  100
12 − 0.7 Pin 23.9
Or RB =
IB
 = 52.5%
RB = 11.3 kΩ



110
www.gradeup.co
Chapter

11 Differential
B
Amplifiers

1. CMRR (Common Mode Rejection Ratio) of a 4. For an op-amp having a slew rate = 2

differential amplifier is defined as the ratio V/μs, the maximum closed loop voltage
gain that can be used when the input
of
signal varies by 0.4 V in10 μsec is ______?
A. Common Mode gain to the Differential
A. 50 B. 100
Mode gain C. 75 D. 25
B. Differential Mode gain to Common 5. For which of the following amplifier circuit

Mode gain with input resistor, R1, and feedback


resistor, Rf, the voltage gain is computed
C. Any one of the above
as:
D. None of the above
 Rf 
1 + 
2. Find differential gain of a dual input  R1 
unbalanced output differential amplifier if A. Inverting amplifier
Rc=20kΩ and rE=2kΩ. B. Non-inverting amplifier
C. Unity follower
A. 1 B. 2
D. Summing amplifier
C. 3 D. 5
6. In the following Circuit, transistor, M1 and
3. For the differential amplifier as shown in M2 has following parameters.
figure find the magnitude of gain of

differential amplifier _______ (Assume VT

=26 mV).

W W
 L  =  L  = 20
 1  2

( VTH )1 = ( VTH )2 = 1V

(K ) = (k )

n
1

n
2
= 100/μA/V2

111
www.gradeup.co

The Voltage V1, V2 and V3 respectively are (Here R1 = 2 kΩ,


A. 1V, 1V, –1.1V R2 = 4 kΩ,
B. 1V, 2V, 1V R4 = 8.6 kΩ,
C. 2V, 1V, 1.32V R3 = 3.3 kΩ)
D. 1V, 1V, -1.32V V1 = 2.2 V and V2 = 1.2 V
7. The open loop gain of an operational A. Circuit represents differential amplifier
amplifier is 10 . An input signal of 1 mV is
5
B. Output voltage Vo = 2.15 V
applied to the inverting input with the non- C. Common mode voltage gain
inverting input connected to the ground. magnitude = 0.2
The supply voltages are ±10V. The output D. Difference mode voltage gain = 2.5
of the amplifier will be? 9. Choose the correct option(s)
A. + 100V B. – 100V A. Differential Gain is the ratio of output
C. + 10 V D. – 10V voltage to differential voltage
MSQs: B. Common voltage is the average sum of
8. For the given figure, consider the OP-AMP to input voltage
be ideal and choose the correct option(s)? C. If gain is 103 and V2 = 5 V and output
voltage is -10 V, then V1 = 5.005 V
D. If gain is 103 and V2 = 5 and output
voltage is -10 V, then common voltage
is 5.005 V

ANSWER

1.B 2. D 3. -140.86 4. A 5. B 6. D 7. D 8. A,B, C,D 9. A,B,D

SOLUTION

1. B. 2. D.

A  For a dual input unbalanced output


CMRR dB = 20log10  d 
 Ac  differential amplifier,
Differential gain Ad= Rc/2rE
Thus, it’s the ratio of Differential mode
Ad = {20x103}/{(2x2) x 103}
gain to the Common mode gain.
Ad=20/4
Ad=5

112
www.gradeup.co

3. Since, Vi = 0, voltage across R1 is V1.


Current through 10K resistor = I10K Hence, using voltage divider,
15 − 0.7 − (−15) R1
I10K = = 2.93mA V1 = V
10 R1 + R f o
Current through Q1 and Q2
Vo  R 
I  voltage gain = = 1 + f 
= 10K = 1.465mA V1  R1 
2
V0 −l 6. D.
= −gmR C = C  R C
Vin VT (VGS)1 = (VGS)2 = –V3
−1.465
=  2500 = −140.86
26
4. A.
V0 V
= ACL  l
t t
V0 / t SR
AcL = =
Vl / t Vl / t

2V /  sec
= = 50
0.4V / 10S
 ACL = 50
Assume FETs are in Saturation using KCL at
5. B. node Voltage V3
A non-inverting amplifier can be ID1 + ID2 = 200A
represented as:
200
ID1 + ID2 = = 100A
2
W
Since   is Same for both M1 and M2 So
L 

k’n are Same for saturation region

kn  W 
( VGS − VTH )
2
ID =  
2 L 

The equivalent circuit for the non-inverting 100  10−6  20


( VGS − 1)
2
 100  10−6 =
amplifier can be drawn as: 2
⇒ (VGS – 1)2 = 0.1
∴ VGS = 1.32V
VD1 = 5-ID1(40 x 103) = 1V
(VDS)Sat= VGS - VTH= 0.32
VGS = VG – VS = 1.32
⇒ 0 – Vs = 1.32
⇒ Vs = -1.32V

113
www.gradeup.co

VDS = VD –VS = 1–[–1.32] = 2.32V Now, Vo = A1V1 + A2V2


(VDS) > (VDS)sat⇒ So the assumption is Vd
V1 = VCM +
true. 2
In the Circuit. V3 = – VGS = – 1.32V Vd
V2 = VCM −
V1 = VD1 = 1V 2
V2 = VD2 = 1V  V   V 
Vo = 2.4  Vcm + d  − 2.6  Vcm − d 
7. D.  2  2
Output voltage of op-amp is: 5
Vo = −0.2Vcm + V
V0 = Av (V+– V-) 2 d
= 105 [0 – 1mV] Vo = –0.2 Vcm + 2.5 Vd
V0 = -100V |Acm| = 0.2
If the output voltage is greater than the |AD| = 2.5
supply voltage, then the op-amp is 9. A, B, D
saturated, so output voltage: Given A = 103
V0 = +Vsat [non-inverting voltage > V2 = 5 V & V1 = –10 V
Inverting voltage] Vo Vo
Ad = =
= – Vsat [inverting voltage > noninverting Vd V2 − V1
voltage] Hence, option A is correct.
So, V0 = - 10 V −10
103 =
8. A, B, C, D 5 − V1
The given circuit iis a difference amplifier V1 = 5.01 V
 R  R V1 + V2
Vo = 1 + 4  V1 − 4 V2 As VC =
 R3  R3 2

 8.6   4  8.6 5.01 + 5


Vo = 1 +  2.2 −  1.2 = = 5.005 V
  
3.3   4 + 2  3.3 2

= 22.4V1 − 2.6V2 Hence, option B is correct.

Vo = 5.28 – 3.13 ≈ 2.15 V



114
www.gradeup.co

115
www.gradeup.co
Chapter

12 Operational Amplifiers

1. Find Vo if 5 mv, 1kHz sinusoidal signal is 4. Find the input resistance in the circuit
applied at input terminal. If R = 10 kΩ and shown
C = 10 μF

A. R1 B. 2R1
C. R1+R2+R3 D. 1
−0.1 5. Consider the OP Amp to be ideal the
A. cos (2π×103t) mV
2  103
voltage at inverting terminal V– is _____ V
B. –0.2π cos (2π × 10 t) volt
3

C. – π cos (2π × 103t) volt



D. − cos (2π × 103t) mV
2
2. For the following circuit. Find the currents
I1& I2? (assuming op – amp to be ideal)

6. Consider the given circuit shown below and


find its cutoff frequency (in KHz)

A. 10.1 mA, 100 μA


B. 100 μA, 10.1 mA
C. 10 μA, 101 μA
D. 1 mA, 10 μA
3. An op–amp has common mode gain of 20
and the value of CMRR is 200. Calculate
the output voltage if the input voltages are
7. The diode has forward voltage drop of V o =
200 μV and 140 μV
0.7V else it is ideal. If Vin is –10V then
A. 243.4 mV B. 234.3 mV
find Vo_____
C. 240 mV D. 236.6 mV

116
www.gradeup.co

10. The value of C required for sinusoidal


oscillation of frequency 1 kHz in the circuit
of fig is

8. The UTP and LTP voltage of the circuit


shown are (assume Vz = 8.3V, Vy = 0.7V).

1
A. F B. 2π μF
2
1
C. F D. 2 6 F
2 6
11. In the circuit shown in fig the op-amp is
ideal. If βF= 60, then the total current
A. 0.8 V and -0.8 V
supplied by the 15 V source is
B. 0.9 V and -0.9 V
C. 0.1 V and -0.1 V
D. 0.75 V and -0.75 V
9. For the transistor in the circuit shown in
figures below, the parameters are β = 100
and VEB(ON) = 0.6 V. The diode is an ideal
zener with VZ = 5.6 V and the op-amp is
ideal. Determine the value of load
resistance RL in kΩ such that the load
A. 123.1 mA B. 98.3 mA
current is a constant. (Take VBC = 0.2 V)
C. 49.4 mA D. 168 mA
12. For the operational amplifier circuit shown
in the figure below, what is the maximum
possible value of R1, if the voltage gain
required is between -10 and -25? (The
upper limit of Rf = 1 MΩ)

A. infinity B. 1 M Ω
C. 100 k Ω D. 40 k Ω

117
www.gradeup.co

13. The output of op Amp V0 (in volts) is, if B)


VBE(ON) = 0.7V and β = 100. Assume op-
amp to be ideal.

C)

14. Recognize the below given circuit?


Functions

P – High pass filter

Q – Amplifier

R – Comparator

S – Low pass filter

A. R - S – P B. R - P - S

A. All pass filter C. Q - S – P D. Q - P - S

B. Non-inverting differentiator 17. Assume that the Op-amps are ideal then
C. Non-inverting integrator find Vo as a function of V1 and V2.
D. Low pass filter
15. The OP Amp circuit shown in figure is

R2ZL
A. A sample and hold circuit A. Vo = ( V − V1 )
R1R3 2
B. An integrator
R1ZL
C. A zero crossing detector B. Vo = ( V − V1 )
R2R3 2
D. A half wave precision rectifier
16. Match the following circuit: R 2ZL
C. V0 =
R1R3 1
( V − V2 )
A)
R1ZL
D. V0 = ( V − V2 )
R2R3 1

18. For the given op – amp circuit having ideal

op – amp circuit. what is the value of

current io?

118
www.gradeup.co

21. Consider the following circuit having switch


S initially closed and is opened at t = 0.
which of the following is correct regarding
output (assume forward drop of Zener to
be 0)

−in −in
A. B.
10 K Rf

−in
C. D. None of these
RL A. It makes a transition from –5V to +5V
19. For the given op – amp circuit. Find the at t = 12.98 μs
common mode rejection ratio(in db). B. It makes a transition from –5V to +5V
Consuming op – amp to be ideal at t = 2.57 μs
C. It makes a transition from +5V to –5V
at t = 12.98 μs
D. It makes a transition from +5V to –5V
at t = 2.57 μs
22. Given transistors Q1& Q2 are identical. The
output voltage at T = 300K is

20. Consider an oscillator circuit having


Schmitt trigger whose voltage
characteristic is given. Find the time period
of vo(t) in msec.

v R 
A. log10  2 1 
 v1R2 

v R 
B. 4.605log10  2 1 
 v1R2 

v R 
C. 2log10  2 1 
 v1R2 
v R 
D. 2303log10  2 1 
 v1R2 

119
www.gradeup.co

23. Consider the capacitive circuit having A. –1A, 2V B. 0A, 2.5V


practical op – amp with input impedance C. –1.25 mA, 2.5V D. 1A, 2V
Rin, and op loop gain Ao1, then what is the 25. For the feedback amplifier as shown the
pole location of this circuit and also find transconductance gm is _____mA/V
the type of the circuit

1  R 
A. − 1 +  , differentiation 26. In the circuit shown in figure, considering
 1  Rin 
Rc 1 +
 Aol  op-amps as ideal, then the value of
V0
is
Vi
1  R 
B. − 1 +  ,interator
RC(1 + Aol)  Rin 

1  R 
C. − 1 +  ,integrator
 1  Rin 
Rc 1 +
 Aol 

1  R 
D. − 1 +  , differentor
RC(1 + Aol)  Rin 
−1 −1
24. Consider the circuit shown below. The A. B.
sCR sC2R2
transistor parameter are Vth = 2.5 V,
−1 −1
Kn = 0.25A/V2 C. 2 2 2
D.
s CR sCR2
(assuming op amp to be ideal and MOSFET
27. For the ideal op-amp circuit shown in
to be in saturation). Find the output
V0
current ID and minimum voltage VDD for figure below, the value of at very high
Vi
MOSFET to be in saturation
frequencies is___?

120
www.gradeup.co

28. The slew rate of an op-amp is 2V/μsec.


The maximum input frequency for an C.
undistorted output is____kHz?

D.

29. Recognize the circuit? 31. Consider the instrumentation amplifier,


with a common mode input voltage of +3V
(DC) and a differential input signal of 80
mV peak sine wave. Find V0?

A. All pass filter B. All stop filter


A. -4.04 sinωt (V)
C. High pass filter D. Band pass filter B. – 6.06 sinωt (V)
30. The transfer characteristics of the circuit C. – 8.08 sin ωt (V)
shown below: D. – 10.10 sin ωt (V)
32. The value of R2 in the regenerative
comparator circuit shown in figure A using
the obtained transfer characteristics as
shown in figure B is?
Figure-A

A.

Figure-B

B.

121
www.gradeup.co

A. 5 k Ω B. 10 k Ω 36. Consider the op-amp in ideal case having


C. 1 k Ω D. 0.5 k Ω V0 = AV1 + BV2
33. The circuit shown is:

A. A low pass filter


A. A = 9.9
B. A clamper
B. A = –9.9
C. A lag compensated inverting amplifier
D. A narrow band video amplifier C. B = 10

34. The following circuit has R = 10 k Ω, D. B = –10

C = 10 μF. The input voltage is a sinusoidal 37. For the following circuit as shown in figure
at 50 Hz. With an RMS value of 10 V. below:
Under ideal conditions, the current Is from
the source is:

A. Magnitude of output and input voltage

is equal

B. Magnitude of output voltage is greater


A. 20π mA lagging by 90°
than the input voltage
B. 20π mA leading by 90°
C. The circuit is equivalent to all pass
C. 10π mA leading by 90°
filter.
D. 10π mA lagging by 90°
D. The circuit is Notch filter.
35. The output of op Amp V0 (in volts) is, if
VBE(ON) = 0.7V and β = 100. Assume op- 38. Consider following options for shown

amp to be ideal. curve:

122
www.gradeup.co

A. Slew rate is maximum rate of change 41. Consider an ideal op-amp circuit as shown
of an op-amp’s output voltage in figure below:
B. Slew rate is minimum rate of change
of an op amp’s output voltage
C. Slew rate is measured in volts per
microseconds.
D. For the curve shown slew rate is 18
V/µsec. Here, open loop gain is 104.
39. Consider a Schmitt Trigger circuit as shown Suppose we want to obtain closed loop
in figure below gain of 10, then which of the following
statement is/are correct?
A. If R1 = 1 then R2 = 9.01 (Both in kΩ)
B. The Ratio of R2/R1 is 9.01
C. The Ratio of R2/R1 is 10.9
D. If R1 = 1 then R2 = 10.9 (Both in kΩ)
42. For the given wein Bridge Oscillator which
of the following combination(s) of R 1 & R2
A. Upper Threshold point is 2 V
is/are correct?
B. Lower Threshold point is –5 V
C. Hysteretic width is 6 V
D. Hysteretic width is 7 V
40. Consider a lossy integrator circuit as shown
in figure below:

A. R1 = 2 kΩ R2 = 4 kΩ
B. R1 = 4 kΩ R2 = 9 kΩ

Suppose at ω = 10 Krad/sec, Peak gain = C. R1 = 6 kΩ R2 =11 kΩ


D. R1 = 9 kΩ R2 = 4 kΩ
20 dB and Gain = 3 dB down from Peak.
43. Choose the correct option(s) regarding the
Then which of the following statement
below circuit?
is/are true?
A. R1 is 1 kΩ
B. R1 is 2 kΩ
C. Rf is 20 kΩ
D. Rf is 10 kΩ

123
www.gradeup.co

A. VA = –0.75 cos 50t B. It’s unit is given by μV/sec


B. VA = –300 cos 50t C. Condition to prevent distortion in op-
C. VO = 15 sin 50t dvo
amp is  slew Rate
D. Vo = –15 sin 50t dt
44. Choose the correct option(s) regarding D. Slew Rate induced distortion may
slow rate of an op-amp? happen in the op-amp if the input
A. It is the maximum Rate at which the signal has rapid changes.
output of an Op-amp can change

ANSWER

1. C 2.B 3.A 4. B 5. 2.4 6. 21.9 7. -10 8. C 9. 12.26 10. A

11.C 12. C 13.5.2 14.B 15. C 16. A 17. C 18. B 19. 41.61 20. 8.1

21.D 22. A 23.B 24. D 25. C 26. C 27. 5 28. 39.8 29. B 30. B

31. C 32. B 33. A 34. D 35. 5.2 36. A,D 37. A,C 38.A,C,D 39. A,C 40. A,D

41.A,B 42. A,B 43.A, C 44. A,B,D

SOLUTION

1. C. d
= – 104 × 10 × 10–6 [5 sin (2π × 103t)
dt
× 10–3]
= – 0.1 × 5 × 10–3 × 2π × 103 cos (2π ×
103t)
∴ Vo = – π cos (2π × 103t) volt.
2. B.
Given Vi = 5 sin (2π × 10 t) × 10
3 –3
volt Draw circuit
& Vo = – Ic × R
dc
Now Ic = C
dt

And Vc = Vi

d i
∴ vo = – RC
dt
Va = Vb = 0 (virtual ground)

124
www.gradeup.co

So 4. B.

1−0
I1 = = 0.1mA
10k

I1' = I1 = 0.1 mA

(assuming ideal op – amp)

VC = 0 – 10 K × 0.1 mA
Applying KVL at input side
VC = – 1V
∴ –V2 + iin × R1 + iin × R1 + V1 = 0
0 − (−1V)
I2 = = 10mA (V2 – V1) = iin × 2R1
100
V2 − V1
I2 = I2’ + I1  = 2R1
iin
I2 = 10.1 mA ∴ input resistance =2R1.

3. A. 5.
Let V– = 6V
Give Acm = 20,
O − 6 6 − V0
CMRR = 200, =
1K 4K
V1 = 200 μV,
Then
V2 = 140 μV
Vo = + 30V
A dm ∵ Vsat = 12V, so Vo can’t be 30 V
now CMRR =
A cm
So, output Vo = +12V.
∴ Adm = CMRR × Acm By KCL at V–
= 200 × 20 O − V− V − − 12
=
∴ Adm = 4000 1K 4K

v + v2 – 4V– = V– – 12
now Vcm = 1
2 5V– = 12
200 + 140 V–=2.4V
=
2
6.
Vcm = 170 μV

Vdm = V1 – V2

= 200 – 140 = 60 μV

Now Vo = Vdm × Adm + (Vcm × Acm)

= (60 × 4000 + 170 × 20) × 10–6

V0 = 243.4 mV

125
www.gradeup.co

So, basically it is a sallen key LPF with 9.

second order and it is having a cutoff Positive terminal of op-amp will be having

frequency Vcc - Vz and using virtual ground concept,


Emitter will have Vcc - Vz. Therefore,
1
f = voltage across emitter will be Vcc - (Vcc -
2 R1C1R2C2
Vz) = Vz
Since,
VZ 5.6
IE = = = 1.12 mA
R1 = R2 = 22K R2 5

C1 = C2 330 PF   100 
I0 = I = (1.12)
1 1 +  E  101 
f =
2 R1R1C1C1 I0 = 1.109 mA
VBC = 0.2 V (Given)
1
= Now simply apply KVL across transistor i.e.
2R1C1
Vcc - Vz - VEB (ON) - VBC - V0= 0
1
= V0= 20 –VZ – 0.6 - 0.2
2  22k  330PF
= 20 – 5.6 – 0.6 - 0.2
f = 21.9 KHz
or V0 = 13.6 V
7. Then,

If Vin = –ve V0 13.6


RL = = = 12.26 k
I0 1.109
Vo' = −ve (Vo’ is more negative than V_)
10. A.
So, diode D1 is ON & D2 is OFF This is Wien-bridge oscillator. The ratio
Equivalent circuit will be R 2 2.1k
= = 2.1 is greater than 2.
R1 1k

So there will be oscillation

So, Vo = –10V (By concept of virtual short)

8. C.

Let Vo=+ve

V0 =9V (because first zener is in forward

bias and other in breakdown region)


1 1
VUTP =V0(R2/R1+R2) =1/10 =0.1 Frequency =  1  103 =
2RC 2(1k)C
VLTP =-V0(R2/R1+R2) = -0.1
1
C= F
2

126
www.gradeup.co

11. C. IE 0.5 0.5


IB = = =
v+ = 5V = v– = vE  + 1 100 + 1 101
The input current to the op-amp is zero. VB = 4+0.7 = 4.7 V
i+15V = iZ + iC = iZ + FiE V0 − 4.7 0.5 V0 − 4.7
IB =  =
15 − 5 60  5  100 101 100
+ = 49.4 mA
47k 61  100 
0.5 = V0 – 4.7
12. C. V0 = 4.7 + 0.5
Rf V0 = 5.2 V
Gain A v = −
R1 14. B.
−R f  
 R1 = 
AV R 
V0 = 1 + V+
1   1 
So, R1 will be    sC  
Av   

So lesser the gain, higher will be R1 Vin R


V0 = (1 + sCR)
1
So, for Av = -10 R+
sC
R1 will be maximum
V0
−R f  = sCR
 R1 = Vin
−10
dVin
R 1  106  V0 (t) = RC
= f = = 105  dt
10 10
∴ Circuit is a non-inverting differentiator.
R1 = 100 k Ω
15. C.
13.
The output of op-amp is connected to
B-E junction is forward biased as V0 it
differentiator. The output of differentiator
positive
is connected to diode. So, the circuit
generates positive pulse. It is a zero-
crossing detector.
16. A.
A. Since, there is no feedback in the
circuit hence, it will act as comparator
∴ V+ = V– = 4V circuit
4 − ( −12 ) B. Given circuit is
 IE = = 0.5 A
32

127
www.gradeup.co

It is practical integrator circuit 17. C.


V0 −Z
∴ Voltage gain = =
Vi R

Here
1 R
Z=R =
sC 1 + sRC
Hence,
V0 [R / 1 + sRC] Applying KVL at input side
=
Vi R ∴ –V2 + iin × R1 + iin × R1 + V1 = 0
V0  1  ∴ (V1 – V2) = – iin × 2R1
= − 
Vi  1 + sRC  V2 − V1
 iin = (i)
From above equation: 2 R1

V0 Now Vo = iout × ZL _________ (ii)


= −1
Vi s −0 Appling KVL across two OP-amps

V0 Iin × R2 + iout × R3 + iin × R2 = 0


=0
Vi s − −2R2
 iout =  iin (iii)
R3
Since, at high frequency gain of op-amp =
0
And gain exists at low frequency. Hence, it From equn (i), (ii) & (iii)
is low pass filter. R2 ZL
 V0 =
R1R3 1
( V − V2 )
C. At low frequency:
At low frequency, capacitor = open circuit 18. B.
Draw circuit again

V0
 =0
Vi

At high frequency
At high frequency capacitor = short circuit
Va = i O R L
For op – amp (2), virtual short is applicable
as negative feedback is there
So, V+ = V– = VK = iORL
Now, in the input loop of op – amp 1
V0
 =1
Vi virtual short is applicable so, current in the
input loop is
Therefore, it is high pass filter.

128
www.gradeup.co

vin − V+ + V− v 11 11
i = = in = =
20K 20K 11 + 1 12

So, V+ for op – amp 1 is v0 v2 11  11 121


A2 =  = =
V+ = Vk – i’ 10kΩ v2 v2 12 12
vin 121
= ioRL −  10K
20K A1 − A2 −10 − 12
Ad = =
vin 2 2
V+ = ioRL − = v−
2 = – 10.04166
Vb = V_+[-i' 10K] ACM = A1 + A2
vin V 121
= ioRL − − in (10 K) = −10 +
2 20 k 12
Vb = ioRL – vin = 0.08333
Since io is flowing through Rf& RL −10 − 04166
CMRR =
So, Vb = io (Rf + RL) 0.0833
⇒ioRL – vin = ioRf + ioRL = 120.49
vin CMRR (dB) = 20 log10 120.99
 io = −
Rf = 41.61 dB
19. 20.

Ad Now, here Vsat = 5V and 0V


CMRR =
ACM And
A1 − A2 It is given that output changes from
Ad =
2 0 → 8V
ACM = A1 + A2 When input is greater than 3V and output
Where A1 = gain due to input 1 changes from 5V → 0
A2 = gain due to input 2
when input is lower than 2v
v −10K  Investing 
now, A1 = 0 = = −10   so, for capacitor, for vo = σv
v1 1  configuaration 
VC(t) = VC(∞) – (VC(∞) – VC(0)) e–t/RC
v
A2 = 0 Where VC (∞) = 8V
v2
VC (o) = 2V
vov
=   2 VC (t) = 3V
v2 v2
3 = 5 – e–t/RC
v0  R   Non −investing  2 = 3 e–t/RC
= 1 + 2   
v2  R1   configration 
2
T1 = −RC ln
= 1 + 10 = 11 3

v2 R4 T1 = 4.05 msec


= (voltage _ devided)
v2 R 4 + R3 Now, for Vo = ov,
VC (∞) = 0

129
www.gradeup.co

VC (o) = 3V −
t
1K 0.01F
vc (+) = 20 − 20 e
VC (t) = 2V
 t
− −5 

= 20 1 − e 10 
 
 
Now, output will move from +5V to –5V
when V– (inverting terminal will exceed the
V+)
And V+ = 4.54V
2 = O + 3 e–t/RC
V– = vc (t)
2
T2 = −RC ln
3  t
− −5 

so, 4.54 = 20 1 − e 10 
T2 = 4.05 msec  
 
Time period = T = T1 + T2 = 8.1 msec
t
21. D. 4.54 − −5
1− = e 10
20
Initially switch is closed and short – circuit
t
so charge at capacitor is zero + = +0.257
10−5
⇒ when switch is closed, then at inverting
t = 2.57 s
terminal, we have ground which basically
will give high positive output (assuming) 22. A.
So, vout = 5V (as the Zener breakdown will Now redraw ckt
occur & give high output)
100k 10
now, V+ = vout =  vout
100k + 10k 11
10
so, initially V+ =  5V
11
= 4.54 V
Now, if we look the input side, then

v1 
i1 =
R1 
 By virtual Ground
v2 
i2 =
R2 
so, here VC (∞) = 20V
Since current in transistor is
VC (0) = OV
v 
And I = I0 exp  BE 
 VT 
−t

vc (+) = vc () − ( vc () − vc (0))


eRC

130
www.gradeup.co

v  So, it,s a integrator circuit with some


i1 = I0 exp  BE1   as both are 
 VT    modification
So,  identical so 
 vBE2    Now, draw the ckt again
i2 = I0 exp   I = Io2 = Io 
 VT   o1
 
i 
 vBE1 = ln  1  VT = 0 − vE1
 Io 
i 
vBE2 = VT ln  2  = o − vE2
 Io 
−333k 333k  333k 
now, vo = vE2 +  1+ V
20k (333 + 20)k  20k  E2

−333 333 353


= vE2 +  v
20 353 20 E1
−333 333 So, vout = AOL (V+ – V–)
= vE2 + v
20 20 E1 vout = – AOL V–
now, apply KCL at node (a)
= 16.65 (VE1 - VE2) vin − v − v v + AoL v −
= − + −
 I I  R Rin 1
= 16.65  VT ln o − VT ln o  sc
 i1 i2 
vin V v
 I i  = − + − + v − (1 + AoL ) sc
= 16.65 × VT  ln  o  2   R Rn R
 
  i1 Io  
vin  1 1 
i  = + + (1 + AoL ) sc  v−
= 16.65 × 0.026 ln  2  R  Rin R 
 i2 
vin
v R  v− =
= 0.4329 ln  2  1   1 1 
R + + (1 + AoL ) sc 
 R2 v1   Rin R 
v R  vout = – AOL V–
= 0.4329 ln  2 1 
 v1 R2  −AoL vin
vout =
Now change loge base to log 10 base  1 1 
R + + (1 + Aol)sc 
v R  R
 in R 
log10  2 1 
= 0.4329  v1 R2  vout Aol
log10 (e) =
vin  R 
 + 1 + R(1 + Aol)sc 
v R  R
 in 
log10  2 1 
 v1 R2  So, pole is at
23. B.  R 
 + 1 + R(1 + Aol) sc = 0
Now, assume that if it is ideal op – amp,  Rin 
then gain will be
 R 
1 − 1 + 
−  Rin 
SC = − 1 s=
R SCR RC(1 + Aol)

131
www.gradeup.co

24. D. 26. C.
Since the op–amp has negative feedback, Step 1: Consider op-amp (1) [A1]
so, virtual ground concept can be used Vi
V0 = − …(1)
here sCR
So, V+ = V– = Vsource = 0 Step 2: Consider op-amp (2) [A2] (VA=Vx)
2RV0 V
VA = Vx = = 0 …(2)
2R + 2R 2
KCL at non-inverting input terminal of A2:

V0 − Vx V − V0
= x + sCVx …(3)
2R 2R
V0 V0 V
V0 − + i
2 = 2 sCR + sCV0 …(4)
VS − (−10V) 2R 2R 2
ID =
10
V0 V V Vi sCV0
0 + 10 − 0 = 0 + 2
+ …(5)
= = 1A 2R 4R 4R 2sCR 2
10
 1 1 1 sC  Vi
ID = kn (VGS – Vth)2  V0  − − −  = …(6)
2 R 4 R 4 R 2  2sCR 2
1 = 0.25 (VG – 0 – 2.5)2
4 = (VG – 2.5)2  sC  Vi
 V0  − = …(7)
± 2 = VG – 2.5  2  2sCR2

VG = 0.5 , 4.5 V0 1
∴ =− 2 2 2
VG = 4.5 V (for gate voltage greater than Vi s CR
threshold voltage)
27.
Now, for saturation
VDS> VGS – Vth
VDD – 0 > VG – 0 – 2.5V
VDD> 4.5 – 2.5
VDD  2V

So minimum VDD is 2V
25. C
From the concept of virtual ground Step 1: KCL at node V2.
V+= V-
Vi V
V- = IoRE VsC
i 1 + = −V0SCf − 0
R1 Rf
I0 1 1
= = 1 + sC1R1  1 + SCfR f 
V − RE 1.5k  Vi   = −V0  
 R1   Rf 
∵ V+ = V (input current of op Amp is zero)
I0 I 1 V0  R  1 + sC1R1 
 = 0 =  10−3 = 0.66 mA/V  = − f  
V− V 1.5 V1  R f   1 + sCfR f 

132
www.gradeup.co

1  30. B.
 + R1C1 
V0 R j Case I:
 =− f  
V1 Ri  1
+ R f Cf 
 j 

As we know that, at very high frequencies,

V0 −R f  R1C1  −C1
=  =
Vi R1  R f Cf  Cf

V0 C 1F
 = 1 = =5
Vi Cf 0.2F When Vin > 0 (inverting terminal > non-
inverting terminal)
V0
=5 So, D1 = ON, D2 = OFF
Vi
Current through
28. Hence, V0 = 0V

 3k  Case II:
V0 = 1 + 2 sin t
 1k 

V0 = 8sinωt

SR = 2πfmaxV0,MAX

2V
= 2fmax (8)
10−6
When Vi < 0, (inverting terminal < non-
→ fmax = 39.8 kHz
inverting terminal)
29. B. D1 = OFF and D2 = ON

 1   −R 
Vin  VA =   Vin = −Vin
 R 
V+ = V−  sC  = Vin
1 1 + sCR
R+  R
sC V0 = 1 +  VA = (2) ( −Vin )
 R
By KCL:
V0 = -2 Vin
Vin Vin V0
Vin − − V0 Hence, = Slope = −2
1 + SCR = 1 + SCR Vin
R 1
sC Therefore, Relation between V0 and Vin i.e.

Vin (sCR) sCRVin Transfer characteristics:


= − V0 (sCR)
1 + sCR 1 + SCR

⇒ V0(sCR) = 0

⇒ V0 = 0 V [All stop filter]

133
www.gradeup.co

31. C. 34. D.

V(A1)non-inverting = 3+ 40 m sin ωt The given equivalent circuit is

V(A2) non-inverting = 3 – 40 m sin ωt

−10 k  10 k  VoutputA 2
V0 = Voutput A1 + 1 + 
10 k  10 k  2

 50k 
= − 1 + [3 + 40m sin t]
 1k / 2 
 
  In ideal op-amp
50k
+ 1 +  [3 − 40 m sin t]
  1k   V + = V-
  2 
   So, V+ = V- = Vs
By applying KCL at inverting terminal,
= – 8.08 sinωt (V)

32. B. V − − 0 V − V0 (s)
+ =0
1/sC R
From the transfer characteristics shown in
or (1 + RCs) Vs = V0(s)
figure B.
At non-inverting terminal
+Vsat = 12V Vs − V0 (s)
Is =
Upper threshold voltage R
or Is = -[RCS/R]Vs
R1
= VuT =
R1 + R2
( +Vszt ) = +4V or Is = - jωCVs
So,
R1 4V 1
 = = Is = 2f  10  10−6  10
R1 + R 2 12V 3

∴ R1 + R2 = 3R1 = 2  50  10−6  10 = 10mA


And ∠Is = -90
∴ R2 = 2R1 = 10 k Ω
Thus, the current from the source is:
33. A.
Is = 10π mA, lagging by 90°
1 35.
For low frequency Zc = 
jC
B-E junction is forward biased as V0 it
−R f positive
So, output. V0 = V
Ri i

We will get the output for high frequency

1
Zc = 0
jC
output , V0  0
∴ V+ = V– = 4V
So, it is a low pass filter
4 − (−12)
 IE = = 0.5A
32

134
www.gradeup.co

1
= 2Vi
1 + sRC
Using equation (1) & (2),
2Vi
Vo = − Vi +
1 + jRC

Vo 1 − jRC
=
IE 0.5 0.5
Vi 1 + jRC
IB = = =
 + 1 100 + 1 101 Taking magnitude
VB = 4+0.7 = 4.7 V Vo 1 + (RC)2
= =1
V0 − 4.7 0.5 V0 − 4.7 Vi 1 + (RC)2
IB =  =
100 101 100
So, |Vo| = |Vi|
0.5 = V0 – 4.7
So, option A is correct.
V0 = 4.7 + 0.5
As |Vo| = |Vi|
V0 = 5.2 V
So, |AV| = 1
36. A, D
Hence, all Pass filter.
Given V0 = AV1 + BV2
So, option C is correct.
−100k
V02 = V2 38. A, C, D
10k
Slew rate is defined as the maximum rate
V02 = –10V2
of change of an op amp’s output voltage
 100K   99K 
V01 = 1 +   (V1 ) and is given units of volts per microsecond
 10K   110K 
(V/µsec).
V01 = 9.9V1
So, option A, C are correct
So, V0 = 9.9V1 – 10V2
For the given curve,
A = 9.9 dVo change in voltage
So Slew rate ( S.R.) = =
B = −10 dt change in time

So, option A and D are correct = Slope of (Vo – t) curve

37. A, C 9 − (−9)
= = 18 V/μsec
V0 = V01 + V02 1
39. A, C
RF
V01 = − V Consider Node ‘X’ at inverting input,
R1 i
Applying KCL at Node X
= ( −R/R ) (V)
i = −Vi ...(1)
Vx − Vin Vx − 2
+ =0
 R  10k 20k
V02 = 1 + F  (Vi )
 R1  2(Vx − Vin ) Vx − 2
+ =0
 1/sC  20k 20k
= 2Vi  
 R + 1/sC  2Vx – 2Vin + Vx – 2 = 0

135
www.gradeup.co

3Vx – 2Vin = 2 Rf
= 10
2Vin + 2 R1
Vx =
3
Rf
If Vo = +10 V R1 =
10
5k
Vref = Vo  At 3 dB down,
5k + 20k
1 A = 20 – 3 = 17 dB
= 10  =2V
5 10
20log10 = 17 dB
If Vo = –10 V 4
1 + [10  R f  0.01  10−6 ]2
5k
Vref = Vo  R f = 10 k
5k + 20k
R1 = 1 k
1
= −10  = −2 V
5 41. A, B
For Vo = –Vsat AOL = 104
Vx > 2 V
Af = 10 ⇒ R2/R1 = ?
2 + 2Vin 6−2
 2  VUTP = Vin  = 2V Vo = AOL Vd
3 2
So, option A is correct. 4  V0  R1 
Vo = 104(V+ – V–) = 10  Vs − 
When vin > 2 V  R1 + R 2 
VUTP = 2 V
Vo 104 (R1 + R 2 )
For Vx < –2 V = = 10 ( A f = 10)
Vs (1 + 104 ).(R1 + R 2 )
Vo = +Vsat
2 + 2Vin (1 + 104)R1 + R2 = 103(R1 + R2)
 −2
3 (1 + 104 – 103)R1 = (103 – 1)R2
6−2 R 2 1 + 104 − 103
VLTP = Vin  − = −4 V
2 = = 9.01
R1 103 − 1
So, option B is incorrect.
VH = VUTP – VLTP = 2 – (–4) = 6 V So if R1 = 1 kΩ

So, option C is correct, and D is incorrect. Then R2 = 9.01 kΩ


40. A, D So options A and B are correct only.
Peak gain 20 dB
42. A & B
at ω =10, 000 rad/sec
We know for the given wein Bridge
Gain is 3 dB down
oscillator
R f /R1
A(dB) = 20log10 R2
14(R f Cf )2 3
R1
R 
20 = 20log10  f  R2
2 R2  2R1
 R1  R1

136
www.gradeup.co

43. A & C 1
Vo = − V dt
Circuit 1 is differentiator for this RC A
L dvi 1 (−0.75 sin50t)
V01 = VA = − Vo = −
R dt 5  5  10 −3
50
VA = −0.75 cos 50t Vo = 15 sin50t

Circuit 2 is integrator 44. A, B & D


Slew Rate unit is V/μsec



137
www.gradeup.co
Chapter

13 Oscillators

1. Assertion (A): All practical RC phase shift 3. Which of the following is/are used for
oscillators generate sinusoidal oscillation generation for radio frequency.
with some amount of amplitude distortion, I. wein–Bridge oscillator
which cannot be avoided. II. LC oscillator
Reason (R): The amplitude distortion of a III. RC oscillator
sinusoidal oscillator is controlled by the IV. crystal oscillator
onset of non-linearity of the amplifying A. I and III only
device. B. II, III and IV only
A. Both A and R are true and R is the C. II and IV only
correct explanation of A D. I, II, III and IV
B. Both A and R are true but R is NOT the 4. Consider the given phase – shift oscillator
correct explanation of A circuit operating at frequency, f = 80 KHz.
C. A is true but R is false The value of resistance RF is _____ kΩ
D. A is false but R is true.
2. The oscillator circuit shown in the figure is:

5. For the given oscillator circuit, find the


type of oscillator, its frequency and
A. Hartley oscillator with foscillation = 79.6 condition for sustained oscillations?
MHz
B. Colpitts oscillator with foscillation = 50.3
MHz
C. Hartley oscillator with foscillation = 159.2
MHz
D. Colpitts oscillator with foscillation = 159.2
MHz

138
www.gradeup.co

1 R2 C R 8. The circuit shown is a sinusoidal oscillator.


weinbridge , = 3 + 3
A. 2 R1R2C3C4 R1 C4 R2
Find the value of resistor Rx (in kΩ) for
oscillation
sustained oscillations.(Let consider the
1 R C R
Weinbridge , 2 = 3 + 4 circuit is at resonance)
B. 2 R3R 4C3C4 R1 C4 R 3
oscillation

1 R2 C3 R 4
phaseshift , = +
C. 2 R3R 4C3C4 R1 C4 R3
oscillation

wein - bridge
D. 1 R2 C R
oscillation , = 3 + 3
2 R3R 4C3C4 R1 C4 R 4

6. Consider the Colpitts oscillator circuit A. 3 B. 3.33

shown in figure below with parameters L = C. 4 D. 4.5

1 μH, C1 = 1 nF, C2 = 1 nF, R = 4 kΩ. 9. Find the relation between R1 and R2 for

What is the oscillator frequency (in MHz, sustained oscillations.

rounded upto 2 decimal values)?

7. Consider the circuit shown in the figure A. R2 = 2R1 B. R2 = 5R1


below: C. R2 = 6R1 D. R2 = 7R1
10. Design the RC elements of a Wein bridge
oscillator as in below figure for operation at
f0 = 10 kHz

R2
For sustained oscillations, the value of
R1

must be equal to:-


A. 1 B. 2
A. 50 kΩ, 159 pF B. 100 kΩ, 159 pF
C. 3 D. 4
C. 150 kΩ, 200 pF D. 20 kΩ, 200 pF

139
www.gradeup.co

11. Consider the following Colpitts’s Oscillator 12. Consider the oscillator as shown in figure
generating 40 kHz frequency. below

For the circuit, which of the following

option is/are correct?


1
A. Output frequency is
2 CeqL eq 1
A. Frequency of oscillation is
 LC
1
B. Output frequency is B. To sustain oscillations Aβ should be 1.
 L eqCeq
C. Rx = 9 kΩ to sustain oscillation.
C. If output Frequency is 40 kHz and C1 =
D. Rx cannot be computed for given
10C2, then C2 = 74 pF
conditions.
D. If output Frequency is 40 kHz and C1 =
10C2 then C2 = 174 pF

ANSWER

1.A 2. B 3. C 4. 236 5. B 6. 7.12 7. B 8. B 9. D 10. B

11. A,D 12. B,C

140
www.gradeup.co

SOLUTION

1. A. 5. B.
Both A and R are correct and R is the It’s a wein – bridge oscillator
correct explanation of A Now,
2. B.
The tank circuit is having two capacitors
and one inductor. So, it is Colpitts
oscillator and frequency is:
1
f =
2 LCeq

C1C2 22
Ceq = = = 1pF = 1pF
C1 + C2 2 + 2
It’s a non – investing configuration, so
1 1  109
f = =
2 10  R 
2 10  10−6  10−12 A = 1 + 2  ( forward gain )
 R1 
f = 50.3 MHz
z3
3. C. = ( feedback gain )
z4 + z3
Wein–Bridge oscillator and RC oscillator
are used for generation of audio frequency 1
where z3 = R 3
sc3
while LC – oscillator and crystal oscillator
are used for generation of radio frequency 1
R3 
sc3
4. =
1
Since it is a phase shift oscillator we know R3 +
sc3
that frequency of oscillation
R3
1 z3 =
f = 1 + sc3R 3
2 6RC
1
R z4 = R 4 +
and F = 29 sc4
R
1 + sc4R 4
It is given that =
sc4
f = 80 KHz
R3
& c = 100 pF
1 + sc3R 3
1 =
so, 80 KHz = R3 1 + sc4R 4
2 6  R  100 pF +
1 + sc3R3 sc4
R = 8.12 kΩ
sc4R 3
R =
Now, f = 29 sc4R 3 + (1 + sc3R 3 ) (1 + sc4R 4 )
R
sc4R3
So, Rf = 29 × (8.12k) =
1 + sc4R3sc3R3 + sc4R 4 + s2c3R3C4R 4
= 236 kΩ

141
www.gradeup.co
Now, for oscillation Aβ = 1 7. B.
  The given circuit is
 R2   sc4R3  =1
1 + 
 R
 ( 2
1   1 + s c4R3 + c3R3 + C4R 4 + s c3R3c4R 4 ) 

Now, to find the frequency put s = jw and


image part equals to ‘0’
 R2   jwc4R 3 
1 +   =1
 R1   1 − w2 ( C3R 3C4R 4 ) + jw (C4R 3 + C3R 3 ) + C4R 4 
 

 
 R2   −wR3c4
1 + 
 =1 Using KCL at the non inverting terminal,

 ( 3 3 4 4 4)
R1   i 1 − w2C R c R − ( c R + C R + C R ) 
3 3 3 4 4 
V0 − Vi V V − V+
(1) = i + i
1 R R
Imaginary sC
1 – w 2 R 3 C 3 R 4 C4 = 0
 2 V
Or V0 (sC) = Vi  sC +  − + -------(1)
1  R R
w=
R3C3R 4C4
Applying KCL at non inverting terminal, we
1 get
f =
2 R 3C3R 4C4
V+ ( V+ − Vi )
+ =0
Now, put this value of w 1 R
sC
in eqn (1) then
 1 V
 R2   wR3C4  or V+  sC +  = i
1 +    =1  R R
 R1   w (C4R3 + C3 + C4R 4 ) 
so, Vi = V+(1 + sRC) …. (1)
R C R + C3R3 + C4R 4
1+ 2 = 4 3 V0(sRC) = V+(1 + sRC) (2 + sRC) - V+
R1 C4R3
= V+[(1 + sRC) (2 + sRC) - 1] … (2)
R C R
1+ 2 =1+ 3 + 4 Therefore, from equations (1) and (2), we
R1 C4 R 3
get the transfer function of the first
R2 C3 R 4
= + network as:
R1 C4 R 3
 R  sRC 
6. T(s) = 1 + 2  
 R1   (1 + sRC)(2 + sRC) − 1 
For Colpitts oscillator, the oscillation
frequency is defined as:  R  sRC 
= 1 + 2  
 R1   2 + 3sRC + s R C − 1 
2 2 2
1
f0 =
2 LCeq  R  sRC 
= 1 + 2   2 2 2 
 R1   s R C + 3sRC + 1 
Where Ceq = C1 ll C2
C1C2 1nF  1nF  R  jRC 
= = = 0.5 nF. or T(j) = 1 + 2   
C1 + C2 1nF + 1nF 
2 2 2
R1  1 −  R C + 3jRC 
So, we get Hence, the condition for oscillation is:
1
f0 =  R   jRC 
2 1  0.5n 1 = 1 + 2   
 R1   3jRC 
f0 = 7.12 MHz

142
www.gradeup.co
R2 11. A, D
The value of is:
R1 L = 100 mH
R2 C1 = 10C2
=2
R1 The output frequency of Colpitts’s
8. B. Oscillator is given by
1
V0 5k f0 =
Gain = A = =1+ 2 L eqCeq
Vf Rx
Given f0 = 40 kHz
β = Vf/V0 = (2k)/(3k+2k) = 2k/5k
So, 40  103 = 1
[LC = short circuit at resonance]
2 (100  10−3 )(Ceq )
For sustained oscillations:
Ceq = 0.0174 × 10–10 F
1
A=
 C1C2
Ceq =
C1 + C2
5k 5k 5k 5 10
∴ 1+ = → = − 1  Rx = k as C1 = 10C2
R x 2k Rx 2 3
So, C2 = 0.1C1
 R x = 3.33 k
So, 0.0174  10−16 = C1C2 =
(10C2 )(C2 )
C1 + C2 (10C2 ) + C2
9. D.
C2 = 174 pF
−R2 V
A= = 0
R1 Vf So, option D is correct.
12. B, C
Vf 1
= =  180
V0 7 Redrawing the oscillator circuit

For sustained oscillations, the condition is


Aβ = 1 < 360°

 R2  1
   180  180 = 1  360
 R1  7 For frequency of oscillations the
admittance of circuit should be only real.
R2 = 7R1
So Img part = 0
10. B.
1 1
Y= + + jC
Using equal values of R and C, we can (2k||2k ) jL
select R = 100kΩ and calculate the  1 
Img part : j  C − =0
required value of C using:  L 
1 1
f0 = C =
2RC L
1
C=
1
=
1 =
( )(
2f0R 6.28 10  103 100  103 ) LC
1
10 −9
f =
= 2 LC
6.28
= 159pF
Therefore, option A is incorrect.

143
www.gradeup.co
For amplifier to sustain oscillators Vf 2k || 2k
= =
Aβ =1 Vout (2k || 2k) + R x + 100
Given A = 10 For Aβ = 1
For β, redrawing circuit at oscillation
 1k 
frequency 10   =1
 1k + R x + 100 
10k = 1k + Rx + 100
Rx = 8.9 = 9 kΩ
So options B and C are correct



144
www.gradeup.co
Chapter
555 Timer
14
43.2
So, 43.2kW = = 12.28 Ton
& Waveform Generators3.5167

1. Pulses of definite width can be obtained 4. Consider the monostable multivibrator


from irregular shaped pulses: circuit shown below. If the monostable
A. When it is given as input to a multivibrator with a 100μs output pulse
monostable multivibrator. then the value of R (in KΩ).
B. When it is given as triggering signal to
a bistable multivibrator.
C. When it is used as input to a schmitt
trigger.
D. When it is used as input to a pulse
transformer.
2. A 555 timer is configured to run in astable
mode. Determine the duty cycle of the
output.

5. For a monostable multi vibrator using IC


555 if RA = 2kΩ, RB = 4 kΩ and C = 0.1 μf
calculate the frequency of output waveform
is ______ KHz
6. The output voltage Vo(t) for the circuit
below is

A . 55.12% B. 72.8%
C. 48.4% D. 32.4%
3. For 555 astable multivibrators, if C = 0.01
μF, RA = 10 kΩ, RB = 50 kΩ, the frequency A. e–t/10 u(t) V
and the duty cycle will be nearly B. –e–t/10 u(t) V
A. 1.6 kHz and 54.5% C. e–t/1.6 u(t) V
B. 1.3 kHz and 54.5% D. – e–t/1.6 u(t) V
C. 1.6 KHz and 46.5%
D. 1.3 kHz and 46.5%

145
www.gradeup.co

7. Find Vo if op-amp is considered ideal? A. 4V to 8V


B. 4V to 6V
C. 2.6V to 4.3V
D. 2.4V to 6V
MSQs:
9. For the given figure, having RA = 6 kΩ, RB

A. -1 V B. +1 V = 3.5 6 kΩ and C = 0.2 μF. Choose the

C. +5 V D. -5 V correct option(s)?

8. Find the range of capacitor voltage VC in


the astable multivibrator given

A. THigh = 0.82 m sec.


B. tlow = 0.48 m seec.
C. Free running frequency = 5570.69 kHz
D. Duty cycle = 73.20%

ANSWER

1. C 2. A 3. B 4. 6.06 5. D 6. A 7. D 8. D 9. B,D

SOLUTION

1. C. 3. B.
Pulses of definite width can be obtained 1
f =
from irregular shaped pulses when it is 0.69 (R A + 2RB ) C
used as input to a schmitt trigger.
1
2. A. =
0.69  (10 + 2  50)  103  0.01  10−6
TON R + R2
Duty cycle = = 1
T R1 + 2 R 2 f = 1.3 KHz
1.3 + 5.7 R A + RB
= & % duty cycle =  100
1.3 + 2  5.7 R A + 2 RB
= 55.12%

146
www.gradeup.co
60 V0(∞) = 0 (∵capacitor would act as open
=  100 = 60%
100 circuit)
 % duty cycle  54.5% Now, time constant of the circuit would be
∴ Option B
 = CR = 8  10−3(1000 + 250) = 10s
4.
So, the expansion for V(t)
Since it is a monostable then time period is
equal to  ( )
V(t) = V0 () + V0 O+ − V0 () e−t / 

T = RC ln3
⇒ V(t) = 0 + (1 – 0) e–t/10
T = 1.1 × R × C
⇒ V(t) = e–t/10 u(t) V
Here, it is given that
7. D.
T = 100 μsec
C = 15 nF
So, 100 T = 1.1 RC
T
R=
1.1  C
100  sec
=
1.1  15  nF
Vp = Vn (due to virtual short)
= 6.06 kΩ
Also Vn – Vo = (0.3mA)*(10K) …………..(1)
5. D.
1 Vo x 20K 2Vo
f0 = Vp = = = Vn
0.69 (R A + 2RB ) C 30K + 20K 5

1 Now from 1 we have


 f0 =
0.69(2 + 2  9)  103  0.1  10−6 2Vo
– Vo = 3
5
∴ fo = 1.45 KHz
6. A. -3Vo = 15

The circuit shown in nothing but a voltage Vo = -5 V

follower circuit. 8. D.

Here, Vo = V– = V+ 2
Vthreshold = 2/3 Vcc =  12 = 8V
So, the simplified circuit becomes 3

1 1
VTrigger = VCC =  12 = 4V
3 3

 12 − 4 
Vc = Vtrigger – (I. 1K) = 4 −    1k
 5k 

= 2.4 V

Vo(0+) = I × (250||1000) Vc= VThreshold – (I.2k)

(As capacitor would act as closed circuit)  12 − 8 


=8−   2k = 6V
 250  1000   4k 
= 5  10−3    = 1V
 250 + 1000  Hence, Vc range is 2.4V to 6V.

147
www.gradeup.co
9. B&D 1.45  103
f=
For the given 555 timer 2.6
tHigh = 0.69 (RA + RB)C f = 557.69 Hz
= 0.69(6 + 3.5)(0.2) ms ton tHigh
D= =
tHigh = 1.31 m sec. ton + t off tHiigh + tlow
tlow = 0.69 RBC = 0.69 × 3.5 × 0.2 ms
1.31
= 0.483 m sec.
D=  73.18%
1.31 + 0.48
1.45 1.45
f =
(R A + 2R B )C (6 + 2  3.5)  0.2  10−3



148
www.gradeup.co

149

Вам также может понравиться